Section 5 Gastroenterology and Hepatology
262
SECTION 5 Gastroenterology and Hepatology
263
CHAPTER 13
264
Ascites
265 Case: A 54-year-old woman with shortness of breath A 54-year-old woman with a remote history of Hodgkin’s
lymphoma treated with mantle-field radiation is admitted to the months. She also describes an enlarging abdomen and bilateral lower began. The patient is an avid bicyclist but has been forced to give it up hasibeen in remission since treatment without evidence of recurrence. hospital with progressive shortness of breath over the past few extremity swelling. She has gained 20 pounds since these symptoms in recent weeks because she can no longer keep up. The lymphoma Soc al history is notable for the consumption of 1 to 2 glasses of wine with dinner on a nightly basis. There is symmetric distention of the abdomen with bulging flanks marks on the anterior chest, consistent with prior radiation therapy. and the presence of shifting dullness to percussion. There are tattoo Neither palmar erythema nor spider angiomas are present. Jugular venous pressure (JVP) is estimated at 16 cm H O and is noted to 2 increase with inspiration. An extra heart sound after S2 is heard best with a phonocardiograph machine (Figure 13-1). Lungs are clear to with the diaphragm of the stethoscope over the apex and is recorded auscultation bilaterally. Hematocrit is 32% and platelet count is 210 K/µL. Aspartate aminotransferase (AST) is 67 U/L, alanine aminotransferase (ALT) is
78 U/L, total bilirubin is 2.3 mg/dL, and international normalized ratio albumin concentration is 2.5 g/dL with a total protein of 4.3 g/dL. (INR) is 1.1. A diagnostic paracentesis is performed, and the ascitic Serum albumin is 3.8 g/dL.
FIGURE 13-1
What is the most likely cause of ascites in this patient?
What is ascites?
What mechanisms maintain peritoneal fluid volume? What are the symptoms of ascites? What physical
Ascites is the abnormal accumulation of fluid within the peritoneal cavity. Normally, the peritoneal cavity contains 25 to 50 mL of serous fluid, which allows the bowels to move around with less friction. Normal intraperitoneal pressure is 5 to 10 mm Hg.1 The balance between fluid entering the peritoneal cavity from the mesenteric capillaries, and fluid absorbed by the lymphatic system normally maintains a constant volume of peritoneal fluid. The maximum absorption capacity of the lymphatic system is 850 mL/d.1 Symptoms of ascites depend on volume, rate of accumulation, and other factors but may include Increased abdominal girth, abdominal fullness, abdominal discomfort, dyspnea, early satiety, and a sense of Decreased mobility.1 Physical findings of ascites may include abdominal distention (usually symmetric), bulging flanks, shifting dullness
266
associated with findings are ascites? What is the role of ultrasonography in the evaluation of ascites?
to percussion, and the presence of a fluid wave.1
Ultrasonography is a bedside imaging modality that is sensitive for detecting the presence of ascites ≥100 mL in volume. Simple ascites is anechoic and appears between the abdominal wall and loops of bowel (Figure 13-2).1
FIGURE 13-2 Abdominal ultrasound (sagittal view) demonstrating loops of bowel and mesentery floating freely within anechoic peritoneal fluid (F). (Courtesy of Philips Medical Systems, Bothell, WA.)
How is the severity of ascites defined? What are the 2 general categories of ascites?
Which laboratory result indicates whether ascites is caused by portal hypertension? What basic fluid characteristics can be helpful in the evaluation of ascites? What is the significance of the gross appearance of ascitic fluid?
What is the significance of the differential cell count in evaluating ascitic fluid? What is the significance of the total protein concentration in ascitic fluid? What is the significance of ascitic fluid glucose and lactate dehydrogenase (LDH) concentrations?
Grade I ascites is ≥100 mL in volume; grade II ascites is ≥1000 mL; and grade III ascites describes a grossly distended abdomen, which indicates the presence of liters of fluid.1 Ascites can be related to portal hypertension or unrelated to portal hypertension.
A diagnostic paracentesis with calculation of the serum-ascites albumin gradient (SAAG) determines whether or not ascites is the result of portal hypertension. The SAAG is calculated by subtracting the concentration of albumin in ascitic fluid from the concentration of albumin in serum (each in g/dL). SAAG ≥1.1 (high gradient) indicates that the fluid is related to portal hypertension, whereas SAAG <1.1 (low gradient) indicates that the fluid is unrelated to portal hypertension.2,3 Routine ascitic fluid evaluation includes gross appearance (eg, color, turbidity, viscosity), total and differential cell counts, total protein concentration, albumin concentration, Gram stain, and culture. Other tests are available and should be used in the appropriate settings (eg, cytology if there is a concern for malignancy).4
The gross appearance of ascitic fluid can help narrow the differential diagnosis. Uncomplicated ascites is typically clear and a pale straw-colored yellow. Hazy, cloudy, or bloody fluid is suggestive of infection. Frank blood indicates hemorrhagic ascites. Milky fluid suggests chylous ascites. Brown fluid can occur in patients with significantly elevated serum bilirubin. However, if the bilirubin concentration of ascitic fluid is greater than that of serum, a ruptured gallbladder or perforated duodenal ulcer should be suspected.4-7 The predominant cell type in ascitic fluid can help narrow the differential diagnosis. For example, a predominance of mononuclear leukocytes (ie, lymphocytes and monocytes) is suggestive of tuberculous ascites. Measurement of the cell count and differential is also necessary to make the diagnosis of spontaneous bacterial peritonitis (SBP), which is defined by the presence of an ascitic polymorphonucleocyte (PMN) count ≥250/µL with or without a positive culture.4 In ascitic fluid, total protein concentration <1.0 g/dL is associated with a higher risk of developing SBP. Total protein concentration is also helpful for differentiating cirrhotic ascites (total protein <2.5 g/dL) from cardiac ascites and other causes of posthepatic portal hypertension (total protein ≥2.5 g/dL).8,9
Ascitic fluid glucose concentration tends to be similar to that of serum except in the setting of peritoneal infection or malignancy, where it is lower as a result of consumption. The LDH concentration of ascitic fluid tends to be higher in patients with infection or malignancy (ascites-to-serum LDH ratio of around 1.0) compared with that in cirrhotics (ascites-to-serum LDH radio of around 0.4).4,10
What ascitic fluid findings are helpful in distinguishing spontaneous bacterial peritonitis from secondary bacterial peritonitis related to bowel perforation into preexisting ascites? What is the significance of Gram stain and culture of ascitic fluid?
What is the significance of cytologic examination of ascitic
Patients with ascites who develop bowel perforation often do not manifest the typical symptoms and signs of a surgical abdomen (eg, rigidity) because the ascitic fluid separates the surfaces of the visceral and parietal peritoneum. In patients with an ascitic fluid PMN count ≥250/µL, secondary bacterial peritonitis from bowel perforation into preexisting ascites is more likely than SBP when any 2 of the following 3 criteria are met: (1) ascitic fluid total protein >1 g/dL, (2) acid fluid glucose <50 mg/dL, and (3) ascitic fluid LDH >upper limit of normal for serum.11,12 Gram stain and culture of ascitic fluid should be included in any infectious workup. There is an Increased diagnostic yield when culture bottles are inoculated at the bedside without delay. Special smears and cultures may be indicated based on other clinical data (eg, acid-fast smear if tuberculosis is suspected).13 Cytologic examination of ascitic fluid can establish a diagnosis of malignancy. Yield is variable and dependent on the type of cancer and the mechanism of ascites formation. Sensitivity approaches 100%
267
fluid? for peritoneal carcinomatosis but is lower for other causes of malignancy-related ascites, such as liver metastases.14,15
268
Ascites Related to Portal Hypertension
What is the fundamental mechanism of ascites related to portal hypertension? What is the gold standard for diagnosing portal hypertension? What are the clinical sequelae of portal hypertension? What are the anatomic subcategories of portal hypertension?
Portal hypertension leads to Increased hydrostatic pressure within the portal capillaries, which favors the movement of fluid out of the capillaries and into the peritoneum.
SAAG ≥1.1 is highly suggestive of portal hypertension. The diagnosis can be confirmed by measuring the hepatic venous pressure gradient (HVPG), which is the difference between wedged hepatic venous pressure (an approximation of portal venous pressure) and free hepatic venous pressure (an approximation of inferior vena cava pressure). A normal HVPG is 3 to 5 mm Hg. Portal hypertension is defined by an HVPG ≥6 mm Hg. Clinically significant portal hypertension occurs when the HVPG is ≥10 to 12 mm Hg. In cases of posthepatic portal hypertension (eg, heart failure), there is a comparable increase in free hepatic venous pressure and wedged hepatic venous pressure such that the HVPG is not elevated.16-18 Portal hypertension can cause splenomegaly with hypersplenism (leading to anemia and thrombocytopenia), portosystemic collaterals, esophageal and gastric varices, ascites, and hepatocellular carcinoma.17,18
Portal hypertension can be prehepatic, hepatic, or posthepatic.
In this framework, the term prehepatic refers to circulation of the portal side of the liver, while posthepatic refers to circulation of the systemic side.
269
Prehepatic Portal Hypertension
What are the prehepatic causes of portal hypertension?
Intraluminal vascular obstruction. Obstruction of the portal system originating outside the vessel walls. Listen for a bruit.
What are the characteristics of portal vein thrombosis?
What condition is most frequently associated with splenic vein thrombosis? What are potential causes of external compression of the portal vein? What are the causes of splanchnic arteriovenous fistula?
Portal vein thrombosis (PVT) and splenic vein thrombosis.
Extrinsic compression of the portal system.
Splanchnic arteriovenous fistula.
Cirrhosis is a significant predisposing factor for PVT; risk increases with disease severity and in those with concomitant hepatocellular carcinoma. PVT also occurs in noncirrhotic patients; often in association with one or more of Virchow’s triad: underlying thrombophilia (eg, primary myeloproliferative disorder), endothelial injury (eg, abdominal trauma), and reduced portal blood flow (eg, external compression by a mass). PVT may be acute or chronic, the latter of which can lead to portosystemic collateral circulation that is visible on imaging. The most common manifestations of acute PVT include abdominal pain, fever, ascites, and variceal bleeding (in patients with coexistent cirrhosis). Chronic PVT is frequently asymptomatic, and the diagnosis is often made incidentally on abdominal imaging. When symptomatic, the most common clinical manifestations of chronic PVT are related to portal hypertension and include variceal bleeding, splenomegaly, anemia, and thrombocytopenia. Intestinal ischemia can be a major complication of both acute and chronic PVT. The initial diagnostic study of choice for acute or chronic PVT is Doppler ultrasonography. Anticoagulation is the cornerstone of medical therapy for patients with acute PVT, and some patients with chronic PVT.19 Chronic pancreatitis is the most common cause of splenic vein thrombosis, accounting for more than one-half of all cases. Splenic vein thrombosis is associated with gastric and esophageal varices. Patients may be asymptomatic. Gastrointestinal bleeding and splenomegaly are the most common clinical manifestations. Splenectomy is indicated in symptomatic patients.20
Portal hypertension can develop as a result of external compression (or invasion) of the portal vein related to tumor (benign or malignant), bulky lymphadenopathy, or hematoma. There may be a role for portal stent placement in some cases.21
Splanchnic arteriovenous fistulae are rare. They are most often acquired as a result of abdominal trauma or spontaneous rupture of splenic artery aneurysms, the latter of which occurs with higher frequency in pregnant women. The increase in portal blood flow related to the shunt leads to a sudden increase in portal pressures.22,23
270
Hepatic Portal Hypertension
What are the hepatic causes of portal hypertension?
271
272
273
274
Posthepatic Portal Hypertension
What are the posthepatic causes of portal hypertension?
An S3 gallop heard over the left lower sternal border. A 42-year-old woman with a history of systemic lupus erythematosus complicated by recurrent bouts of acute pericarditis presents with dyspnea and is found to have elevated JVP, Kussmaul’s sign (inspiratory increase in JVP), and ascites. Vascular structures.
Which cause of right-sided heart failure often presents with a pulsatile liver?
How common is ascites in patients with constrictive pericarditis?
What classic triad of clinical manifestations is associated with Budd-Chiari syndrome?
What are the causes of inferior vena cava obstruction?
Right-sided heart failure. Constrictive pericarditis.
Thrombosis of the hepatic vein (Budd-Chiari syndrome) and obstruction of the inferior vena cava (IVC).
Congestive hepatopathy and portal hypertension can develop in patients with right-sided heart failure from various causes, including cor pulmonale, valvular disease, ischemic disease, and cardiomyopathy. Severe tricuspid regurgitation results in anterograde blood flow from the right ventricle to the right atrium, which is transmitted to the venous system, producing characteristic physical findings. Lancisi’s sign (CV fusion wave) can be visualized in the jugular venous waveform in the neck, and a pulsatile liver can be palpated in the abdomen. For a video of Lancisi’s sign, see the associated reference.30 Ascites occurs in almost one-half of patients with constrictive pericarditis. The most common physical finding in these patients is elevated JVP, which serves as an important clue that ascites may be primarily driven by cardiac disease rather than liver disease.31 Acute Budd-Chiari syndrome classically presents with abdominal pain, ascites, and hepatomegaly. Chronic Budd-Chiari syndrome is often asymptomatic but can be associated with the sequelae of portal hypertension. Most cases are associated with underlying thrombophilia, usually a primary myeloproliferative disorder (eg, polycythemia vera). Doppler ultrasonography of the liver is the initial diagnostic test of choice. Without treatment, Budd-Chiari syndrome is associated with poor prognosis. Anticoagulation is the cornerstone of treatment and significantly improves survival rates.32 IVC obstruction can occur as a result of thrombosis related to underlying thrombophilia, extrinsic compression by tumor, Infective phlebitis, and regional inflammation from trauma or surgery. It may be acute or chronic. Abdominal pain, ascites, and lower extremity edema are typical clinical manifestations. Chronic cases are more insidious due to the development of collateral circulation. Doppler ultrasonography is the initial diagnostic test of choice. Treatment depends on the underlying cause but includes anticoagulation, procedures such as balloon dilation and stenting, and surgery.33
275
Ascites Unrelated to Portal Hypertension
When portal hypertension is absent, what are the 2 main types of ascitic fluid produced?
Ascitic fluid unrelated to portal hypertension can be protein-poor (total protein generally <2.5 g/dL) or protein-rich (total protein generally ≥2.5 g/dL). Ascitic fluid should not be categorized as protein-poor or protein-rich before first determining that it is unrelated to portal hypertension based on the SAAG.34
276
Protein-Poor Ascites Unrelated to Portal Hypertension
What fundamental mechanism underlies the development of protein-poor ascites unrelated to portal hypertension? What is the principal contributor to capillary oncotic pressure?
Decreased oncotic pressure within the portal capillaries favors the movement of protein-poor fluid out of the capillaries and into the peritoneum. Large nonfiltered proteins, primarily albumin, contribute the bulk of capillary oncotic pressure.35
What are the causes of ascites related to low capillary oncotic pressure?
Foamy urine. An impoverished child with a protuberant abdomen. Increased fecal α-1 antitrypsin clearance.
What are the general treatment strategies for ascites related to nephrotic syndrome? What pediatric condition is associated with severe protein-calorie malnutrition and protuberant abdomen?
What is protein-losing enteropathy?
Nephrotic syndrome. Protein-calorie malnutrition.
Protein-losing enteropathy.
In addition to addressing the underlying disease, patients with ascites caused by nephrotic syndrome tend to respond to salt restriction and diuretics. This is in contrast to most other causes of ascites unrelated to portal hypertension, in which salt restriction and diuretics have unpredictable results.36
Severe protein-calorie malnutrition can result in kwashiorkor, a condition with clinical manifestations such as lethargy, dermatitis, thinning hair, protuberant abdomen, and lower extremity edema (Figure 13-4). Abdominal distention in these patients is primarily related to hepatomegaly from fatty infiltration, with or without ascites. Severe protein- calorie malnutrition is common in patients with cirrhosis, where it contributes to morbidity and mortality, including the development of refractory ascites.37,38
FIGURE 13-4 A child with kwashiorkor. Notice the thinning hair, protuberant abdomen, and lower extremity edema. (From Ferrier DR. Lippincott’s Illustrated Reviews: Biochemistry. 6th ed. Philadelphia, PA: Lippincott Williams & Wilkins; 2014.)
Protein-losing enteropathy is a rare condition that develops when protein is lost in the Gastrointestinal tract, resulting in hypoalbuminemia, which leads to peripheral edema, ascites, and pleural and pericardial effusions. It can be caused by a variety of conditions, most commonly cardiac (eg, after the Fontan procedure for congenital single ventricle) or Gastrointestinal (eg, inflammatory bowel disease) in nature. The initial diagnostic test of choice is measurement of fecal clearance of α-1 antitrypsin, which is elevated in patients with protein-losing enteropathy. Treatment for this condition is aimed at the underlying cause, but a high-protein diet is generally recommended. 39
277
Cirrhosis can contribute to low capillary oncotic pressure (see Figure 13- 3). However, despite the mixed nature of the fluid, it is typically associated with elevated SAAG and is therefore classified as a cause of ascites related to portal hypertension.34
278
Protein-Rich Ascites Unrelated to Portal Hypertension
What mechanisms underlie the development of protein-rich ascites unrelated to portal hypertension?
When portal hypertension is absent, protein-rich ascites develops via one or more of the following mechanisms: Increased capillary permeability, Increased fluid production, and impaired fluid reabsorption from lymphatic obstruction.
What are the causes of protein-rich ascites unrelated to portal hypertension?
May be Intraperitoneal malignancy. discovered on cytologic analysis of peritoneal fluid. Alcohol and Pancreatitis. gallstones account for most cases. An infection that Tuberculosis. most often involves the upper lobes of the lungs (see Figure 31-3). A 49-year-old man Hemoperitoneum. with cirrhosis develops hypotension and an acute drop in hematocrit after a paracentesis procedure. Milky ascites. Chylous ascites. A 44-year-old Myxedema. woman presents with weight gain, constipation, and cold intolerance. Peritoneal signs Perforated viscus. are present on abdominal examination.
What are the Malignant ascites is indicative of peritoneal carcinomatosis: the presence of malignant cells in the peritoneal cavity. It characteristics of accounts for about 10% of all cases of ascites and is most commonly associated with lymphoma, peritoneal malignant ascites? mesothelioma, and ovarian, uterine, colorectal, pancreatic, lung, and breast cancers. Several mechanisms are responsible for malignant ascites related to peritoneal carcinomatosis, including altered capillary permeability and lymphatic obstruction, both of which contribute to the accumulation of peritoneal exudate. Prognosis is poor, with most patients surviving only weeks to months after the diagnosis is made. Malignancy, particularly lymphoma, is a common cause of chylous ascites, resulting from lymphatic infiltration and obstruction. When malignancy-related ascites occurs as a result of massive liver metastases, it is often protein-poor and associated with a high gradient (SAAG ≥1.1).4,15,40 What is pancreatic Pancreatic ascites is an uncommon complication of pancreatic disease that refers to the accumulation of pancreatic ascites? fluid within the peritoneal cavity as a result of pancreatic ductal disruption or pseudocyst leakage. It most commonly occurs in men with alcohol-related chronic pancreatitis who are between 20 and 50 years of age, but can also occur as a result of abdominal trauma or ampullary stenosis. Ascitic fluid is protein-rich (typically >3 g/dL) with elevated amylase (typically >1000 U/L). Conservative management includes bowel rest, somatostatin analogues, and large-volume paracentesis, repeated as necessary. In some patients, more invasive options may be necessary, including pancreatic duct stent placement and surgery.41
279
studies are available for tuberculosis? peritoneal
What is chylous ascites?
What are the causes of hemoperitoneum?
How common is myxedema ascites?
What are the characteristics of ascites caused by viscus perforation?
commonly the lungs. However, only one-third of patients with peritoneal tuberculosis have clinical or radiographic evidence of pulmonary disease. Ascitic fluid is protein-rich (typically >3 g/dL) with an elevated total cell count with a lymphocytic predominance (>70%). Acid-fast bacilli smear is positive in <3% of cases; culture yield is higher, but results can take weeks to return. Ascitic fluid adenosine deaminase levels are useful; however, false negatives occur in patients with concurrent HIV infection or cirrhosis, and false positives occur in malignant ascites. Ascitic fluid polymerase chain reaction for Mycobacterium tuberculosis has been shown to be highly sensitive and specific. Laparoscopy may reveal characteristic macroscopic findings (eg, thickened peritoneum with tubercles), and peritoneal biopsy yields a histologic diagnosis in most cases. 42-44 Chylous ascites describes the presence of thoracic or intestinal lymphatic fluid within the peritoneal cavity that occurs as a result of disruption to the lymphatic system, usually from trauma or obstruction. Chylous ascites has a creamy or milky appearance and is rich in triglycerides (>200 mg/dL). A wide range of conditions can result in chylous ascites, including cirrhosis, malignancy (particularly lymphoma), infection, and trauma. In the industrialized world, cirrhosis and malignancy account for the majority of cases; in the rest of the world, tuberculosis and filariasis are more common.45 In the general population, hemoperitoneum most commonly occurs as a result of abdominal trauma or nonmalignant gynecologic conditions. In patients with preexisting ascites, hemorrhagic ascites is defined by an RBC count >50,000/ µL (hematocrit 0.5%). It most often develops spontaneously or secondary to iatrogenic abdominal trauma from diagnostic or therapeutic procedures. Spontaneous hemorrhagic ascites in cirrhotics usually develops insidiously without hemodynamic instability or clinical evidence of bleeding. However, acute and massive hemoperitoneum (with hematocrit values that approach 40%) can occur with rupture of hepatocellular carcinoma or an intra-abdominal varix, requiring prompt surgical management. Abdominal trauma (eg, injury to the inferior epigastric artery during paracentesis) can also result in acute and massive hemoperitoneum.5 Ascites is a rare complication of hypothyroidism; however, hypothyroidism is a common condition. Myxedema ascites is thought to develop as a result of Increased capillary permeability. The fluid is virtually always protein-rich (typically ≥2.5 g/dL). However, the SAAG is sometimes unexpectedly Increased (≥1.1) despite the absence of portal hypertension. Myxedema ascites resolves with thyroid replacement therapy.46,47 In addition to causing secondary peritonitis in patients with preexisting ascites, viscus perforation can cause de novo ascites. Fluid is usually blood-tinged with an elevated white blood cell count. The combination of a PMN count ≥250/ µL and a polymicrobial Gram stain or culture is suggestive of bowel perforation (SBP is polymicrobial in only 10% of cases). The combination of a PMN count <250/µL and a polymicrobial Gram stain or culture (ie, polymicrobial bacterascites) is suggestive of a traumatic paracentesis during which the needle entered the bowel.48,49
280 Case Summary A 54-year-old woman with a remote history of Hodgkin’s lymphoma treated with mediastinal radiation presents with dyspnea
and is found to have ascites, elevated JVP, and the presence of an extra heart sound.
What is the most likely cause of ascites in this patient? Constrictive pericarditis.
281 Bonus Questions
less likely thanfconstrictive “cardiac cirrhosis”).tHowever, the preservation of hepatic synthetic function and the absence of characteristic physical findings (eg, spider angiomas) wn as Which clinical eatures in Cirrhosis is the mos common cause of ascites, and the patient in this case does consume significant amounts of alcohol. However, the elevated JVP is this case make cirrhosis suggestive of right-sided heart failure or constrictive pericarditis. Hepatic fibrosis may develop during the later stages of congestive hepatopathy (kno pericarditis? suggest that advanced liver disease is not present. 50 What are the significant The ascitic fluid in this case is associated with a high gradient (SAAG ≥1.1), indicating that it is related to portal hypertension. In this setting, elevated ascitic characteristics of the fluid total protein ≥2.5 g/dL is suggestive of cardiac ascites, which can be related to right-sided heart failure or constrictive pericarditis, or other posthepatic ascitic fluid in this case? causes of portal hypertension (eg, Budd-Chiari syndrome).9 cause of constrictiveely? constrictive pericarditis, often many years after treatment. When the history is unavailable or unreliable, the presence of tattoo markers on the skin can beia What is the most lik As part of the treatment for Hodgkin’s lymphoma, the patient in this case received external beam radiation therapy to the mediastinum, which can result n pericarditis in this case helpful clue to previous external beam radiation therapy. What is the most likely Extra heart sounds that occur near S2 include split S2, S3 gallop, opening snap, and pericardial knock. The extra sound in this case (see Figure 13-1) is most source of the extra heart likely a pericardial knock, based on location, pitch, and clinical history. The pericardial knock is thought to arise from the abrupt cessation of early diastolic sound in this case? ventricular filling because of noncompliant pericardium. It tends to occur earlier than the S3 gallop and is higher in pitch. 51 What is the significance of Paradoxical rise in JVP with inspiration, known as Kussmaul’s sign, is suggestive of impaired filling of the right ventricle. It is a common finding in patients venous pressure with the change in jugular with constrictive pericarditis. For a video of Kussmaul’s sign, see the associated reference.52 inspiration in this case? diagnostic studies canihelp Hemodynamic assessment withiright and lefticardiac catheterization may demonstrate characteristic findings of constriction, including abrupttand rapid n. What additional In patients with constrictive per carditis, card ac computed tomography or magnetic resonance imaging may reveal pericardial thickening and calcificatio confirm the diagnosis n early diastolic filling, elevated and equal end-diastolic pressures in all 4 chambers, and discordance between the peak inspiratory pressures of he right this case? ventricle compared with the left. 50 What options arelavailable Conservative management of ascites relatedlto constrictive pericarditis includesidietary restriction, diuretic medication, and large-volume paracentesis, constrictive pericarditis? to treat ascites re ated to which can be repeated as necessary. Surgica pericardiectomy is the only definit ve treatment for constrictive pericarditis. 50
282 Key Points
Ascites is the abnormal accumulation of fluid within the peritoneal cavity. Symptoms of ascites include Increased abdominal girth,
abdominal fullness, abdominal discomfort, dyspnea, early satiety, Physical findings of ascites include abdominal distention, bulging and a sense of Decreased mobility. flanks, shifting dullness to percussion, and the presence of a fluid wave.
Ascites can be related to portal hypertension (SAAG ≥1.1) or Portal hypertension can be prehepatic, hepatic, or posthepatic.r unrelated to portal hypertension (SAAG <1.1). Ascites unrelated to portal hypertension can be protein-poor o protein-rich. Ascitic fluid analysis, including gross appearance, cell count and differential, total protein, Gram sta n and culture, cytology, and
other ancillary tests, can be helpfuliin determining the etiology of ascitesiincludes salt restriction, diuretics, and large-volument of ascites. In add tion to addressing the underlying cause, manageme paracentesis.
283
References 1. Moore CM, Van Thiel DH. Cirrhotic ascites review: pathophysiology, diagnosis and management. World J Hepatol. 2013;5(5):251-263. Pare P, Talbot J, Hoefs JC. Serum-ascites albumin concentration gradient: a physiologi
- approach to the differential diagnosis of ascites. Gastroenterology. 1983;85(2):240-244.c
- The serum-ascites albuminfgradient is superiorlto the exudate-transudate concept in. Runyon BA, Montano AA, Akriviadis EA, Anti lon MR, Irving MA, McHutchison JG the differential diagnosis o ascites. Ann Intern Med. 1992;117(3):215-220.
-
Huang LL, Xia HH, Zhu SL. Ascitic fluid analysis intthe differential diagnosis of 5. Akriviadis EA. Hemoperitoneum in patients with ascites. Am J Gastroenterol. ascites: focus on cirrhotic ascites. J Clin Transl Hepa ol. 2014;2(1):58-64. 1997;92(4):567-575.
-
Chinnock B, Hendey GW. Can clear ascitic fluid appearance rule out spontaneous bacterial peritonitis? Am J Emerg Med. 2007;25(8):934-937.
-
Runyon BA. Ascitic fluid bilirubin concentration as a key to choleperitoneum. J Clin Gastroenterol. 1987;9(5):543-545.
-
RunyonlBA. Low-protein-concentration ascitic fluid is predisposed to spontaneous 9. Runyon BA. Cardiac ascites: a characterization. J Clin Gastroenterol. 1988;10(4):410- bacteria peritonitis. Gastroenterology. 1986;91(6):1343-1346. 412.
-
Runyon BA, Hoefs JC. Ascitic fluid chemical analysis before, during and after spontaneous bacterial peritonitis. Hepatology. 1985;5(2):257-259.
-
Akriviadis EA, Runyon BA.iUtility.of an algorithm in.differentiating spontaneous 12. Runyon BA, Hoefs JC. Ascitic fluid analysis in the differentiation of spontaneous from secondary bacterial per tonitis Gastroenterology 1990;98(1):127-133. bacterial peritonitis from Gastrointestinal tract perforation into ascitic fluid.
Hepatology. 1984;4(3):447-450. 13. Runyon BA, Antillon MR, Akriviadis EA, McHutchison JG. Bedside inoculation of blood culture bottles with ascitic fluid is superior to delayed inoculation in the
detection of spontaneous bacterial peritonitis. J Clin Microbiol. 1990;28(12):2811-2812. 14effusion. A retrospective study of cytopathologic diagnoses with autopsy confirmation. . DiBonito L, Falconieri G, Colautti I, Bonifacio D, Dudine S. The positive peritoneal Acta Cytol. 1993;37(4):483-488.
-
Runyon BA, Hoefs JC, Morgan TR. Ascitic fluid analysis in malignancy-related ascites. Hepatology. 1988;8(5):1104-1109.
-
Pinzani M, Rosselli M, Zuckermann M. Liver cirrhosis. Best Pract Res Clin Gastroenterol. 2011;25(2):281-290.
-
Berzigotti A, Seijo S, Reverter E, Bosch J. Assessing portal hypertension in liver diseases. Expert Rev Gastroenterol Hepatol. 2013;7(2):141-155.
-
Bari K, Garcia-Tsao G. Treatment of portal hypertension. World J Gastroenterol. 2012;18(11):1166-1175.
-
Chawla YK, Bodh V. Portal vein thrombosis. J Clin Exp Hepatol. 2015;5(1):22-40. 20. Sakorafas GH, SarriMG, Farley DR, Farnell MB. The significance of sinistral portal 21stenosis in a patient with malignant tumor and Gastrointestinal bleeding. Int Surg. hypertension compl cating chronic pancreatitis. Am J Surg. 2000;179(2):129-133. . Sakurai K, Amano R, Yamamoto A, et al. Portal vein stenting to treat portal vein
284
2014;99(1):91-95. 22. Johnston GW, Gibson JB. Portal hypertension resulting from splenic arteriovenous fistulae. Gut. 1965;6(5):500-502. . Siablis D, Papathanassiou ZG, Karnabatidis D, Christeas N, Katsanos K, Vagianos C. Splenic arteriovenous fistula and sudden onset of portal hypertension as complication
23of a ruptured splenic artery aneurysm: successful treatment with transcatheter arterials embolization. A case study and review of the literature. World J Gastroenterol.
2006;12(26):4264-4266. 24. Elbaz T, Esmat G. Hepatic and intestinal schistosomiasis: review. J Adv Res. 2013;4(5):445-452.
-
Navasa M, Garcia-Pagan JC, Bosch J, et al. Portal hypertension in acute liver failure. Gut. 1992;33(7):965-968.
-
Selmi C, Bowlus CL, Gershwin ME, Coppel RL. Primary biliary cirrhosis. Lancet. 2011;377(9777):1600-1609. . Lindor KD, Kowdley KV, Harrison ME, American College of Gastroenterology. ACG
27Clinical Guideline: primary sclerosing cholangitis. Am J Gastroenterol. 2015;110(5):646- 659; quiz 60.
- Schouten JN, Garcia-Pagan JC, Valla:DC, Janssen HL. Idiopathic noncirrhotic portal 29occlusive disease: current situation and perspectives-a position statement from the hypertension. Hepatology. 2011;54(3) 1071-1081. . Mohty M, Malard F, Abecassis M, et al. Sinusoidal obstruction syndrome/veno- European Society for Blood and Marrow Transplantation (EBMT). Bone Marrow
Transplant. 2015;50(6):781-789. 30. Mansoor AM, Mansoor SE. Images in clinical medicine. Lancisi’s sign. N Engl J Med. 2016;374(2):e2.
-
Howard JP, Jones D, Mills P, Marley R, Wragg A. Recurrent ascites due to constrictive pericarditis. Frontline Gastroenterol. 2012;3(4):233-237.
-
Darwish Murad S, Plessier A, Hernandez-Guerra M, et al. Etiology,:management, and outcome of the Budd-Chiari syndrome. Ann Intern Med. 2009;151(3) 167-175.
- Srinivas BC, Dattatreya PV, Srinivasa KH, Prabhavathi, Manjunath CN. Inferior vena cava obstruction: long-term results of endovascular management. Indian Heart J.
2012;64(2):162-169. 34theiascites total protein concentration in separation of “transudative” and “exudative” 35. Weisberg HF. Osmotic pressure of the serum proteins. Ann Clin Lab Sci. . Rector Jr WG, Reynolds TB. Superiority of the serum-ascites albumin difference over asc tes. Am J Med. 1984;77(1):83-85. 1978;8(2):155-164.
- Runyon BA. Management-of adult patients with ascites caused by cirrhosis. Hepatology. 1998;27(1):264 272.
- Eghtesad S, Poustchi H, Malekzadeh R. Malnutrition in liver cirrhosis:the influence of protein and sodium. Middle East J Dig Dis. 2013;5(2):65-75. . Tierney EP, Sage RJ, Shwayder T. Kwashiorkor from a severe dietary restriction in an
388-month infant in suburban Detroit, Michigan: case report and review of the literature. Int J Dermatol. 2010;49(5):500-506.
- Umar.SB, DiBaise JK. Protein-losing enteropathy: case illustrations and clinical 40pathophysiology and therapeutic measures. World J Gastrointest Surg. 2012;4(4):87-95. review Am J Gastroenterol. 2010;105(1):43-49; quiz 50. . Sangisetty SL, Miner TJ. Malignant ascites: a review of prognostic factors,
285
- Kanneganti K, SrikakarlapuditS, Acharya B, Sindhaghatta V, Chilimuri S. Successfulc management of pancreatic asci es with both conservative management and pancreati duct stenting. Gastroenterol Res. 2009;2(4):245-247. . Hillebrand DJ, Runyon BA, Yasmineh WG, Rynders GP. Ascitic fluid adenosine
42deaminase insensitivity in detecting tuberculous peritonitis in the United States. Hepatology. 1996;24(6):1408-1412.
-
Mimidis K. Peritoneal tuberculosis. Ann Gastroenterol. 2005;18(3):325-3295. 44from 11 cases and review of the literature. World J Gastroenterol. 2004;10(24):3647-e . Uzunkoy A, Harma M, Harma M. Diagnosis of abdominal tuberculosis: experienc 3649.
-
Cardenas A, Chopra S. Chylous ascites. Am J Gastroenterol. 2002;97(8):1896-1900. 46. de Castro F, Bonacini M, Walden JM, Schubert TT. Myxedema ascites. Report of two cases and review of the literature. J Clin Gastroenterol. 1991;13(4):411-414.
-
Ji JS, Chae HS, Cho YS, et)al. Myxedema ascites: case report and literature review. J Korean Med Sci. 2006;21(4 :761-764.
- Cappell MS. Intestinal (mesenteric) vasculopathy. I. Acute superior mesenteric i. arteriopathy and venopathy. Gastroenterol Clin North Am. 1998;27(4):783-825, v
-
RunyonrBA, Hoefs JC, Canawati HN. Polymicrobial bacterascites. A unique entity in 50. Kwan DM, Dhaliwal G, Baudendistel TE. Thinking inside the box. J Hosp Med. the spect um of infected ascitic fluid. Arch Intern Med. 1986;146(11):2173-2175. 2008;3(1):71-76.
-
Marriott HJL. Bedside Cardiac Diagnosis. Philadelphia, PA: Lippincott Company; 1993. 52. Mansoor AM, Karlapudi SP. Images in clinical medicine. Kussmaul’s sign. N Engl J Med. 2015;372(2):e3.
286
CHAPTER 14
287
Cholestatic Liver Injury
288 Case: A 48-year-old woman with pruritis 8 monthsiof progressive fatigue and generalized itching. The fatigueh gardening and going on walks. The itching, which involves the whole never smoked cigarettes, and does not consume alcohol regularly or A prev ously healthy 48-year-old woman presents to the clinic wit has prevented her from activities she previously enjoyed, such as body, has gotten worse over the past few months. The patient has use illicit drugs. Urine and stool have been normal in quality. Vital signs are unremarkable. The abdomen is nontender, and the orange papules and nodules over the extensor surfaces of both elbows, liver is normal in size. Murphy’s sign is negative. There are yellow- and around the eyelids (Figure 14-1). Aspartate aminotransferase (AST) is 52 U/L; alanine
aminotransferase (ALT), 71 U/L; total bilirubin, 1.8 mg/dL (direct bili-range 30-120 U/L); γ-glutamyl transferase (GGT), 290 U/L (reference ratio (INR), 0.9. Transcutaneous abdominal ultrasound shows normal rubin, 1.4 mg/dL); alkaline phosphatase (ALP), 340 U/L (reference range 2-30 U/L); albumin, 3.6 mg/dL; and international normalized liver size and contour and no biliary dilation.
FIGURE 14-1 (From Elder DE, Elenitsas R, Rubin AI, et al. Atlas and Synopsis of Lever’s Histopathology of the Skin. 3rd Edition. Philadelphia, PA: Lippincott Williams & Wilkins; 2012.)
What is the most likely cause of cholestatic liver injury in this patient?
What biochemical laboratory pattern describes cholestatic liver injury?
What demographic groups tend to have higher serum alkaline phosphatase levels at baseline? What are the sources of alkaline phosphatase in the body?
When serum alkaline phosphatase levels are elevated, what other laboratory result is suggestive of a hepatic source? What is the role of alkaline phosphatase in hepatic metabolism?
Diseases of the liver produce characteristic biochemical patterns of injury. Distinguishing cholestatic liver injury from hepatocellular liver injury can be helpful in narrowing the differential diagnosis. Cholestatic liver injury refers to the predominance of serum ALP elevation compared with serum aminotransferases. Serum bilirubin levels are usually also elevated.1,2 Blacks tend to have slightly higher serum ALP values than whites; smokers have slightly higher values than nonsmokers; and pregnant women have higher values than nonpregnant women.2 In addition to the hepatobiliary system, ALP is present in bone, intestine, kidney, placenta, and white blood cells. Diseases of the hepatobiliary system and bone are the most common causes of elevated serum ALP.2 When serum ALP is elevated, concomitant elevation of either serum GGT or serum 5′-nucleotidase (5′-NT) is suggestive of hepatobiliary disease. The combination of elevated serum ALP and normal serum GGT or 5′-NT should shift attention from the liver to the bone or elsewhere. Fractionating serum ALP isoenzymes can aid in identifying the source but is usually not necessary.2 ALP is an enzyme found on the surface of cells that transports metabolites across the membrane. In the liver, it is found on the surface of bile duct epithelia. Cholestasis and accumulating bile salts augment the synthesis and release of ALP, resulting in Increased serum levels.1,3,4
289
Compared with the
aminotransferases, why are serum alkaline phosphatase levels typically late to rise in the setting of acute liver injury and slow to fall after it has resolved? How does bilirubin end up in bile?
Is cholestasis associated with elevated serum levels of unconjugated or conjugated bilirubin? What are the clinical manifestations of cholestasis?
What are the 2 general categories of cholestatic liver injury?
What is the anatomic distinction between intrahepatic and extrahepatic cholestasis? What is the fundamental mechanism of extrahepatic cholestasis? What first-line imaging modality is used to distinguish extrahepatic from intrahepatic cholestasis?
What additional information can be obtained with computed tomography (CT) imaging in a patient with cholestatic liver injury? If transcutaneous abdominal ultrasonography and CT imaging are negative for biliary duct dilation, what additional imaging modalities can be used to detect the presence of extrahepatic cholestasis? What is the gold standard for evaluation of the biliary tract?
necrosis, ALP is synthesized in response to cholestasis and accumulating bile salts, which requires Unlike the aminotransferases, which are immediately leaked into the circulation upon hepatocyte additional time. This makes ALP late to rise when there is acute liver injury; its long serum half-life (around 1 week) makes it slower to peak and fall after liver injury resolves.1
Water-insoluble unconjugated (indirect) bilirubin is a product of hemoglobin breakdown within the reticuloendothelial system. It travels to the liver where it is converted to water-soluble conjugated (direct) bilirubin, which is then excreted into bile.1 Cholestasis predominantly results in elevated conjugated bilirubin, which typically occurs when the liver has lost at least half of its excretory capacity.1
Clinical manifestations of cholestasis depend on the underlying cause but may include pruritis, fatigue, dark-colored urine, acholic (pale or light-colored) stools, jaundice, and palpable dilation of the gallbladder (Courvoisier sign).2 Cholestatic liver injury can occur as a result of extrahepatic or intrahepatic cholestasis.
Intrahepatic causes of cholestasis occur at the hepatocellular level and generally result in impaired bilirubin transport from hepatocytes to the bile canaliculi and intrahepatic ducts. Extrahepatic causes of cholestasis involve the larger bile ducts. Extrahepatic cholestasis occurs when large bile ducts become obstructed, either intrinsically or by external compression, resulting in impaired excretion of bile into the small intestine. Transcutaneous abdominal ultrasonography (TUS) can be helpful in differentiating extrahepatic and intrahepatic cholestasis. Biliary duct dilation is suggestive of an extrahepatic process (ie, biliary obstruction) but typically does not identify the cause. The overall sensitivity of TUS is limited, particularly in obese patients. Furthermore, biliary duct dilation may be entirely absent in some cases of obstruction (eg, partial obstruction). Insignificant biliary duct dilation unrelated to obstruction can be seen in patients with a history of cholecystectomy.2,5 CT imaging has similar sensitivity compared with TUS for identifying biliary duct dilatation, but it can provide additional information about the liver parenchyma and may be more helpful in identifying mass lesions (eg, pancreatic tumor).2
Given the limited sensitivities of TUS and CT imaging, negative studies in patients strongly suspected of extrahepatic cholestasis should be followed by further imaging, including magnetic resonance cholangiopancreatography (MRCP) or endoscopic ultrasound (EUS), which are significantly more sensitive.2
ERCP is the gold standard for evaluating the biliary tract and can identify the cause and level of obstruction. It offers additional diagnostic information via brushings and biopsies as well as therapeutic interventions such as stone extraction, sphincterotomy, dilation of strictures, and stent placement.2
290
Extrahepatic Cholestasis
What life-threatening condition can develop in patients with extrahepatic cholestasis? What are the 2 anatomic subcategories of extrahepatic cholestasis?
What is the anatomic relationship between the extrahepatic biliary tree and the pancreas?
Extrahepatic cholestasis can lead to ascending cholangitis, a life-threatening infection of the biliary tract. Charcot’s triad of fever, right upper quadrant abdominal pain, and jaundice is present in most patients.2 Obstruction of the large bile ducts can occur as a result of processes associated with the biliary system or the adjacent pancreas.
In most patients, the common bile duct passes posterior to the head of the pancreas on its way to the duodenum, becoming at least partially covered by pancreatic tissue. This is referred to as the intrapancreatic portion of the common bile duct (Figure 14-2).6
FIGURE 14-2 Anatomy of the extrahepatic biliary tree and its relationship to the pancreatic head. (From Mulholland MW, et al. Greenfield’s Surgery: ScientificlPrinciples and Practice. 6th ed. Philadelphia, PA: Wolters Kluwer Hea th; 2017.)
291
Extrahepatic Cholestasis Related to the Biliary System
What are the biliary causes of extrahepatic cholestasis?
292
293
Cholelithiasis and acute cholecystitis are not usually associated with cholestatic liver injury because biliary obstruction is limited to the gallbladder. However, a minority of patients may experience a complication known as Mirizzi syndrome in which a gallstone in the neck of the gallbladder or cystic duct causes external compression of the adjacent common hepatic duct. This leads to biliary obstruction proximal to the common hepatic duct along with a cholestatic pattern of liver injury.10
294
295
Extrahepatic Cholestasis Related to the Pancreas
What are the pancreatic causes of extrahepatic cholestasis?
A 38-year-old man with active alcohol abuse presents with nausea, vomiting, and epigastric abdominal pain that radiates to the back. A 46-year-old man with a history of recurrent acute pancreatitis has chronic abdominal pain, weight loss, and pancreatic parenchymal calcifications on cross-sectional imaging. A late complication of acute pancreatitis, typically developing over the course of a few weeks. A 60-year-old man with a heavy smoking history presents with painless jaundice and a palpable gallbladder.
What are the mechanisms of cholestasis caused by acute and chronic pancreatitis?
What are pancreatic pseudocysts?
Acute pancreatitis.
Chronic pancreatitis.
Pancreatic pseudocyst.
Pancreatic cancer.
In the setting of acute pancreatitis, tissue edema or abscess can compress the intrapancreatic portion of the common bile duct; in chronic pancreatitis, compression of the duct occurs as a result of encroaching pancreatic fibrosis. Patients with chronic pancreatitis often experience recurrent cholestasis during episodes of acute pancreatitis when a partially compressed duct becomes further compromised by tissue edema. This can eventually lead to common bile duct stricture. Surgical intervention should be considered for patients with persistent cholestasis related to chronic pancreatitis.6 Pancreatic pseudocysts arise when there is disruption of the pancreatic duct, resulting in leakage of pancreatic fluid into the surrounding tissues that develops an encapsulating fibrous wall over time. Pseudocysts occur in up to one-fifth of patients with acute pancreatitis but also occur in patients with chronic pancreatitis. Most pseudocysts resolve spontaneously, although rupture can occur, resulting in pancreatic ascites, and the potential for the formation of fistulae to other viscera. Pseudocysts can also cause extrahepatic biliary obstruction from extrinsic common bile duct obstruction (Figure 14-4). Patients with biliary obstruction should be considered for cyst drainage.6,11
FIGURE 14-4 Enhanced CT image through the pancreatic head shows a mature pancreatic pseudocyst (P). Note the distended gallbladder (G), compressed duodenum (straight arrow), and distended common bile duct (curved arrow). Several small pancreatic calcifications (arrowheads) are seen adjacent to the pseudocyst, indicative of chronic pancreatitis. (From Pope TL, Harris JH. Harris & Harris’ The Radiology ofiEmergency Medicine. 5th ed. Philadelphia, PA: Lippincott Williams & W lkins; 2012.)
What are the risk factors The strongest risk factors for pancreatic cancer are family history (lifetime risk for a patient with ≥3 affected
296
for pancreatic cancer? first-degree relatives is about 40%), advanced age, cigarette smoking, and a history of chronic pancreatitis. Cancers that arise near the head of the pancreas present with obstructive jaundice in most cases; other manifestations include weight loss, abdominal pain, and new-onset diabetes. Prognosis is poor, with 5-year survival rates <10%. 12
297
Intrahepatic Cholestasis
What are the main Most patients with chronic intrahepatic cholestasis experience fatigue and pruritis. Hypercholesterolemia is clinical features of common, often producing xanthomas (cholesterol deposits in tendon sheaths and bony prominences) and chronic intrahepatic xanthelasma (cholesterol deposits around the eyelids).2 cholestasis? What diagnostic Once extrahepatic cholestasis has been ruled out with imaging, investigation into intrahepatic causes of cholestasis studies are helpful should begin. Occasionally, the cause of intrahepatic cholestasis is identified during the initial workup (eg, in the workup of hepatocellular carcinoma detected on initial imaging). Otherwise, history and physical examination should direct a intrahepatic serologic workup. Liver biopsy may ultimately be necessary in some cases.2 cholestasis? What are the general Intrahepatic cholestasis can be obstructive, toxic, or infectious. mechanisms of intrahepatic cholestasis?
298
Intrahepatic Cholestasis Related to Obstruction
What are the obstructive causes of intrahepatic cholestasis?
May be associated with elevated serum α- fetoprotein levels. More common in women, with a 10:1 female-to-male ratio. A 28-year-old man with ulcerative colitis is found to have elevated serum ALP and GGT levels. Obstructive pulmonary disease, pancreatic insufficiency, and infertility. These conditions commonly affect multiple organs; low-voltage electrocardiogram is a classic cardiac manifestation, and kidney enlargement is a classic renal manifestation. Red blood cells lodged within the hepatic sinusoids. A patient develops cholestasis after liver transplantation. A patient develops cholestasis after bone marrow transplantation.
What are the malignant causes of intrahepatic cholestasis? What serologic test(s) should be obtained to evaluate for primary biliary cholangitis?
How is the diagnosis of primary sclerosing cholangitis made?
What is the mechanism of intrahepatic cholestasis caused by cystic fibrosis?
What are the features of hepatic sarcoidosis?
Hepatocellular carcinoma.
Primary biliary cholangitis (PBC).2
Primary sclerosing cholangitis.
Cystic fibrosis.
Infiltrative disorders (eg, sarcoidosis).
Sickle cell anemia.
Posttransplant complications.
Graft-versus-host disease (GVHD).
The most common malignant causes of intrahepatic cholestasis include hepatocellular carcinoma, cholangiocarcinoma, lymphoma, and metastases from primary tumors of the Gastrointestinal tract (eg, colon), breast, lung, and pancreas.13,14 Positive serum antimitochondrial antibody (AMA) is the hallmark of PBC, with sensitivity and specificity that exceed 95%. Additional antibody testing (eg, anti-Sp100 and anti-Gp210) and liver biopsy are required to make the diagnosis in the few patients who are AMA-negative. Some patients may have an overlap syndrome, with features of PBC and autoimmune hepatitis. A subset of these patients has clinical features of PBC but a serologic pattern suggestive of autoimmune hepatitis (eg, negative AMA and positive antinuclear antibody, with or without positive anti–smooth muscle antibody).2,15 PSC is associated with the presence of perinuclear anti–neutrophil cytoplasmic antibodies (p-ANCA). MRCP is the noninvasive diagnostic study of choice for PSC, and may demonstrate diffuse multifocal stricturing and dilations of the intrahepatic and/or extrahepatic ducts. ERCP has the advantage of offering additional diagnostic capabilities via brushings to rule out cholangiocarcinoma and therapeutic capabilities (eg, biliary dilation and stenting).2 The cystic fibrosis transmembrane regulator (CFTR) protein is found within biliary epithelium and is partly responsible for the water and ion composition of bile. In patients with severe CFTR mutations, Increased bile viscosity leads to congestion and obstruction within the intrahepatic bile ducts. Chronic biliary obstruction results in fibrosis and eventually cirrhosis. 16 Most patients with systemic sarcoidosis have hepatic involvement, however the majority are asymptomatic. A cholestatic pattern of liver injury may be the only clue to the diagnosis. CT imaging of the liver may reveal hepatomegaly or multiple hypodense nodular lesions. Histology reveals noncaseating granulomas along the portal tract. However, noncaseating granulomas are not specific for sarcoidosis and other confirmatory
299
What are the features of sickle cell intrahepatic cholestasis?
What are the causes of intrahepatic cholestasis after liver transplantation?
What are the features of graft-versus-host disease of the liver?
features should be sought, such as additional extrahepatic manifestations of sarcoidosis (eg, pulmonary disease). Glucocorticoids are the treatment of choice. Other infiltrative diseases that cause cholestasis include lymphoma and amyloidosis.2,13 Sickle cell intrahepatic cholestasis is likely a severe variant of sickle hepatic crisis, which is a syndrome characterized by right upper quadrant abdominal pain, fever, jaundice, leukocytosis, and moderate elevation of ALP. It is caused by hepatic ischemia related to widespread sinusoidal sickling. Patients with sickle cell intrahepatic cholestasis often develop severe jaundice, renal impairment, a bleeding diathesis, and encephalopathy. A more common cause of cholestasis in patients with sickle cell anemia is choledocholithiasis from pigmented gallstones related to chronic hemolysis.17 Intrahepatic cholestasis is common after liver transplantation. Early causes (<6 months posttransplant) include ischemia-reperfusion injury, initial graft dysfunction, immunosuppressive medication, infection, and acute cellular rejection. Late causes include hepatic artery thrombosis, chronic rejection, biliary complications (eg, stricture at an anastomotic site), recurrent cholestatic disease (eg, PBC or PSC), recurrent viral hepatitis, and posttransplant lymphoproliferative disorder.18 GVHD occurs following hematopoietic stem cell transplantation when donor T cells identify and attack host antigens as foreign. In the liver, the small bile duct cells are the targets of this immune-mediated attack. It is usually accompanied by GVHD of the skin or Gastrointestinal tract resulting in the typical presentation of rash, diarrhea, and elevated ALP.13
300
Intrahepatic Cholestasis Related to Toxicity
What are the toxic causes of intrahepatic cholestasis?
This iatrogenic cause of liver injury can present with a variety of different biochemical patterns of injury, including hepatocellular, cholestatic, or mixed. Increased estrogen levels likely play a role in the pathogenesis of this condition. Associated with mild hepatocellular liver injury (aminotransferase levels typically <300 U/L) with an AST:ALT ratio ≥2:1. Used to provide nutrition to patients who cannot tolerate enteral nutrition.
Medication (ie, drug-induced liver injury).
Intrahepatic cholestasis of pregnancy.2
Alcoholic hepatitis.19
Total parenteral nutrition (TPN).
How common is drug- Drug-induced liver injury is the most common cause of acute liver failure in the industrialized world, induced liver injury? responsible for more than one-half of all cases. A detailed history of prescription and nonprescription medications, herbs, supplements, and other substances, is a critical part of making the diagnosis. The timing of liver injury after the onset of exposure is widely variable and drug-dependent, ranging from hours to up to a year. Cholestatic injury tends to recover slowly after discontinuation of the offending agent. However, some cases progress to chronic liver disease.20 Why is the serum Intrahepatic cholestasis of pregnancy typically occurs around 25 to 32 weeks of gestation. Affected women alkaline phosphatase experience pruritis that is usually worse at night. Serum aminotransferases and bilirubin may be elevated. level of limited Because the placenta is a source of ALP, serum levels may be elevated in pregnant women even in the absence diagnostic utility in of cholestasis, limiting its diagnostic value. An elevated serum bile acid level is a better biochemical marker for evaluating for intrahepatic cholestasis of pregnancy. Resolution after delivery is expected; however, recurrences during intrahepatic cholestasis subsequent pregnancies are common. Ursodeoxycholic acid can be used to manage symptoms during of pregnancy? pregnancy.2 What are the clinical Patients with alcoholic hepatitis have a compatible history of alcohol use, fever, and tender hepatomegaly. Most features of alcoholic patients experience modest elevations in aminotransferases (usually <300 U/L), marked hyperbilirubinemia, hepatitis? and variable elevations in ALP (from normal to values in the thousands U/L). Alcoholic hepatitis is usually classified as a cause of hepatocellular liver injury, but it can be cholestatic in some cases.13,19,21 In what time frame does In adults, cholestasis typically occurs several weeks after starting TPN but may occur later. Infusions with high cholestasis develop after lipid content are more likely to cause cholestasis. Discontinuation of TPN or modification of the infusion starting total parenteral formula (eg, reduction in lipid emulsion component) usually results in resolution of cholestasis. However, nutrition? cholestasis may continue to progress to severe liver disease in a minority of patients.2
301
Intrahepatic Cholestasis Related to Infection
What are the infectious causes of intrahepatic cholestasis?
A common cause of cholestasis in the Sepsis. intensive care unit. Most commonly associated with Viral hepatitis. marked hepatocellular liver injury, often with aminotransferase elevation
50 times the upper limit of normal. A 58-year-old man who recently Hepatic tuberculosis (TB). emigrated from India presents with fever, right upper quadrant abdominal pain, shortness of breath, and cough and is found to have hepatomegaly and a cholestatic pattern of liver injury.
Which organisms are most commonly associated with the cholestasis of sepsis? Is cholestasis an early or late finding in the course of acute viral hepatitis?
What diagnostic tests are available for hepatic tuberculosis?
Cholestasis from sepsis most commonly develops in association with gram-negative organisms and some gram-positive organisms, such as Staphylococcus aureus and Streptococcus pneumoniae. Other bacteria associated with cholestasis include Leptospira, Clostridium, and Borrelia.13 Cholestasis tends to follow the acute phase of viral hepatitis (which is associated with hepatocellular liver injury), usually coinciding with clinical improvement. It most commonly occurs with hepatitis A, acute hepatitis C, and hepatitis E infections. Jaundice may last for up to 6 months, but the vast majority of patients experience complete recovery. Most cases of acute viral hepatitis are diagnosed during the acute phase of the illness, at which time there is a hepatocellular pattern of liver injury.13 Hepatic TB should be suspected in any patient with risk factors for TB (eg, homelessness) who presents with hepatomegaly, fever, respiratory symptoms, and cholestatic liver injury. CT scan of the liver can identify findings suggestive of either focal or miliary hepatic TB. The most specific diagnostic test for hepatic TB is liver biopsy with histology (Figure 14-5) and mycobacterial culture. However, sensitivity is poor. Polymerase chain reaction (PCR) for Mycobacterium tuberculosis of the biopsy specimen is highly sensitivity and specific.22
FIGURE 14-5 Hepatic TB. Normal liver cells surround a granuloma, which is comprised of a central core of necrotic tissue and a halo of lymphocytes and macrophages, some of which fuse to form multinucleated giant cells. (From Chowdhury SH, Cozma AI, Chowdhury JH. Essentials for the Canadian Medical Licensing Exam. 2nd ed. Philadelphia, PA: Wolters Kluwer Health; 2017.)
302
303 Case Summary fatigue and pruritisland is found to have cholestatic liver injury ic A previously hea thy 48-year-old woman presents with chron without evidence of biliary dilation on imaging.
What is the most likely cause of cholestatic liver injury in this patient? Primary biliary cholangitis.
304
Bonus Questions
What is significant The absence of biliary duct dilation on TUS in this case is suggestive of an intrahepatic cholestatic process. When suspicion for extrahepatic cholestasis remains about the despite a negative TUS or CT, patients should undergo further diagnostic imaging studies (eg, MRCP, EUS, or ERCP). Otherwise, history and physical transcutaneousasound examination should direct a workup for intrahepatic cholestasis.2 abdominal ultra results in this c se? What is significant Fatigue and pruritis are the most common symptoms in patients with chronic intrahepatic cholestasis caused by conditions such as PBC.2 about the fatigue and pruritis in this case? What is significantngs The skin lesions described over the elbows in this case are consistent with xanthomas, and the lesions around the eyelids (see Figure 14-1) are consistent with What is the of the age PBC is a condition found primarily in women, with a female-to-male ratio of 10:1. Patients most frequently present in the fifth or sixth decade of life.2,15 in this case? about the skin findi xanthelasma. Both of these lesions are suggestive of hyperlipidemia, which is found in most patients with PBC. 15 significance and gender of the patient in this case? What is the nextn this The chronicity of symptomsiand absence oflbiliary dilation on TUS in this case make extrahepatic cholestasis (ie, biliary obstruction) unlikely. The history and for primary biliaryent general population.iHowever, just under one-half of all patients with PBC have an unsatisfactory response to ursodeoxycholic acid. Patients-with progressive 5 diagnostic step i physical examination are suggestive of PBC. Serum AMA titer should be checked. AMA is positive in the vast majority of patients with PBC and is associated case? with specificity >95%. The combination of e evated ALP and positive AMA is diagnostic of PBC. Additional antibody testing (eg, anti-Sp100 and anti-Gp210) and/or liver biopsy are requ red to make the diagnosis in the few patients who are AMA-negative. 2,15 What is the treatm Ursodeoxycholic ac d is the treatment of choice for PBC and is capable of halting disease progression, leading to an overall survival rate that is similar to the cirrhosis? PBC may develop cirrhosis and portal hypertension, and in some cases require liver transplantation. PBC recurs after transplantation in one quarter of cases. 1
305 Key Points
Cholestatic liver injury refers to the predominance of serum ALP elevation compared with serum aminotransferases. Serum bilirubin levels are usually also elevated.
Diseases of the hepatobiliary system and bone are the most Elevated serum GGT or 5′-NT levels can confirm a hepatobiliary cause but include pruritis, fatigue, dark-coloredlurine, acholic g The presence of dilated biliary ducts on imaging suggests Extrahepatic cholestasis occurs as a result of obstruction of the common causes of elevated serum ALP. source of ALP elevation. Clinical manifestations of cholestasis depend on the underlyin stools, jaundice, and palpable dilation of the gal bladder. Cholestasis can be extrahepatic or intrahepatic. extrahepatic cholestasis. larger bile ducts related to either the pancreas or the biliary system.
Intrahepatic cholestasis can be obstructive, toxic, or infectious.
306
References 1. Giannini EG, Testa R, Savarino V. Liver enzyme alteration: a guide for clinicians. CMAJ. 2005;172(3):367-379.
-
Siddique A, Kowdley KV. Approach to a patient with elevated serum alkaline phosphatase. Clin Liver Dis. 2012;16(2):199-229. Moss DW. Physicochemical and pathophysio ogical factors in the release of
-
membrane-bound alkaline phosphatase fromlcells. Clin Chim Acta. 1997;257(1):133- 140. Schlaeger R, Haux P, Kattermann R. Studies on the mechanism of the increase i
-
alkaline phosphatase activity in cholestasis: significance of the hepatic bile acidn serum concentration for the leakage of alkaline phosphatase from rat liver. Enzyme.
1982;28(1):3-13. 5. Scharschmidt BF, Goldberg HI, Schmid R. Current concepts in diagnosis. Approach to the patient with cholestatic jaundice. N Engl J Med. 1983;308(25):1515-1519.
- Skellenger ME,lPatterson D, Foley NT, Jordan Jr PH. Cholestasis due to compression of the common bi e duct by pancreatic pseudocysts. Am J Surg. 1983;145(3):343-348.
- Lim JH. Liver flukes: the malady neglected. Korean J Radiol. 2011;12(3):269-279.
-
Sripa B, Kaewkes S, Sithithaworn P, et al. Liver fluke induces cholangiocarcinoma. PLoS Med. 2007;4(7):e201.
-
Das AK. Hepatic and biliary ascariasis. J Glob Infect Dis. 2014;6(2):65-72. 10cholecystocholedochal fistula, and gallstone ileus. Am J Gastroenterol. 2002;97(2):249- . Abou-Saif A, Al-Kawas FH. Complications of gallstone disease: Mirizzi syndrome, 254.
-
Crino SF, ScalisifG, Consolo P, et al. Novel endoscopic management for pancreatic pseudocyst with istula to the common bile duct. World J Gastrointest Endosc. 2014;6(12):620-624.
-
Yabar CS, Winter JM. Pancreatic cancer: a review. Gastroenterol Clin North Am. 2016;45(3):429-445.
-
Assy N, Jacob G, Spira G, Edoute Y. Diagnostic approach to patients with cholestatic jaundice. World J Gastroenterol. 1999;5(3):252-262.
-
Sica GT, Ji:H, Ros PR. CT and MR imaging of hepatic metastases. Am J Roentgenol. 15. Carey EJ, Ali AH, Lindor KD. Primary biliary cirrhosis. Lancet. 2015;386(10003):1565- 2000;174(3) 691-698. 1575.
-
Strazzabosco M. Transport systems in cholangiocytes: their role in bile formation and cholestasis. Yale J Biol Med. 1997;70(4):427-434.
-
Banerjee S, Owen C, Chopra S. Sickle cell hepatopathy. Hepatology. 2001;33(5):1021- 1028.
-
Corbani A, Burroughs AK. Intrahepatic cholestasis after liver transplantation. Clin Liver Dis. 2008;12(1):111-129, ix.
-
Lucey MR, Mathurin P, Morgan TR. Alcoholic hepatitis. N Engl J Med. 2009;360(26):2758-2769.
-
Padda(MS, Sanchez M, Akhtar AJ, Boyer JL. Drug-induced cholestasis. Hepatology. 21. Singal AK, Kodali S, Vucovich LA, Darley-Usmar V, Schiano TD. Diagnosis and 2011;53 4):1377-1387. treatment of alcoholic hepatitis: a systematic review. Alcohol Clin Exp Res.
307
2016;40(7):1390-1402. 22tuberculosis with considerations in human immunodeficiency virus co-infection. BMC . Hickey AJ, Gounder L, Moosa MY, Drain PK. A systematic review of hepatic Infect Dis. 2015;15:209.
308
CHAPTER 15
309
Diarrhea
310 Case: A 58-year-old man with dyspnea A previously healthy 58-year-old man presents to the clinic with watery, occurs up to 15 times per day, continues during periods of persistent diarrhea over the course of 6 months. The diarrhea is time when he does not eat, and is sometimes associated with abdominal cramping. There is no blood in the stools. He has experienced a weight loss of 20 pounds over this time. He has no
experienced night sweats or fever. He complains of intermittent t episodes of redness and warmth of his face and neck that last around
5 minutes. Over the past 4 weeks, he has become increasingly short of United States. He does not use any medications, including laxatives. breath with activity. There has been no recent travel outside of the He has not taken antibiotics in years. The patient is cachectic. The liver is palpable 4 finger widths below stool. There is a 2/4 decrescendo diastolic murmur that augments with the costal margin. Digital rectal examination reveals loose, brown inspiration, best heard over the left upper sternal border. A fecal leukocyte test is negative. Stool osmotic gap is <50 hydroxyindoleacetic acid (5-HIAA) is elevated. Cross-sectional tic Whole-body somatostatin-receptor scintigraphy (octreotide scan) is mOsm/kg. Measurement of 24-hour urinary excretion of 5- imaging of the abdomen shows numerous ring-enhancing hepa lesions of varying size, some with central necrosis (Figure 15-1A). completed (Figure 15-1B).
FIGURE 15-1
What is the most likely cause of diarrhea in this patient?
What is diarrhea?
How much fluid normally enters the Gastrointestinal (GI) tract in a 24-hour period?
Diarrhea describes an increase in water content, volume, or frequency of stool, typically occurring at least 3 times in a 24-hour period. Acute diarrhea lasts ≤14 days; persistent diarrhea lasts >14 days; and chronic diarrhea lasts >30 days. The duration of a diarrheal illness can narrow the differential diagnosis. For example, most cases of acute diarrhea are infectious in nature.1,2 Each day, approximately 7.5 liters of fluid enters the GI tract via oral ingestion and secretions. Most of this fluid is reabsorbed by the small intestine, leaving about 1.2 liters to enter the colon each day. The colon typically recovers about 1 liter of fluid (with a reserve absorptive capacity of up to 3 L/d), leaving a stool volume of <200 mL/d in healthy individuals. Large-volume diarrhea is suggestive of a small bowel process.3
311
How does the Gastrointestinal tract absorb water?
What are the 4 general mechanisms of diarrhea?
Absorption of water within the GI tract is coupled with ion transport. Passive and active transport of ions such as sodium, chloride, and bicarbonate in the small and large intestines generate electrochemical gradients that drive the absorption of water. Diarrhea is often associated with electrolyte disturbances (eg, hypokalemia) and non-anion gap metabolic acidosis.3,4 Diarrhea can be inflammatory, osmotic, secretory, or related to intestinal dysmotility.
Most etiologies of diarrhea act through a variety of these mechanisms. For simplicity, entities will be listed under 1 category of the framework in this chapter.
312
Inflammatory Diarrhea
What is the fundamental mechanism of inflammatory diarrhea? What historical features can be suggestive of inflammatory diarrhea?
What stool study is suggestive of inflammatory diarrhea?
What life-threatening complication of inflammatory colitis should be considered in patients with abdominal distention?
Inflammatory diarrhea occurs when mucosal disruption results in exudation of serum into the intestinal lumen; destruction of the absorptive epithelium also leads to malabsorption.3,5 Inflammatory diarrhea caused by either invasive infections or noninfectious conditions may be associated with abdominal pain, fever, tenesmus, and bloody or mucoid stools. Extraintestinal manifestations (eg, uveitis) may be present in some conditions, such as inflammatory bowel disease (IBD).1,3 Inflammatory diarrhea caused by either invasive infections or noninfectious conditions may be associated with the presence of polymorphonuclear leukocytes or leukocyte proteins (eg, calprotectin, lactoferrin) in stool. Evidence of an inflammatory response is frequently absent in patients with noninvasive infectious diarrhea.1 Patients with inflammatory colitis related to either infectious or noninfectious conditions can develop toxic megacolon, a life-threatening complication that presents with abdominal distention and signs of systemic toxicity, such as fever, tachycardia, and delirium. Diagnosis can be confirmed with abdominal imaging (Figure 15-2).6
FIGURE 15-2 Upright abdominal radiograph of a patient with ulcerative colitis presenting with fever, abdominal pain, and distention showing marked dilatation of the colon, consistent with toxic megacolon. The transverse colon measures more than 10 cm in diameter (arrowheads). (From Brant WE, Helms C. Fundamentals of Diagnostic Radiology. 4th ed. Philadelphia, PA: Lippincott Williams & Wilkins; 2012.)
313
Noninvasive Infectious Diarrhea
What are the 3 main types of organisms that cause noninvasive infectious diarrhea?
Noninvasive infectious diarrhea is most commonly caused by viruses, bacteria, or protozoa.
314
Noninvasive Viral Diarrhea
How is the diagnosis The diagnosis of viral gastroenteritis is usually made on clinical grounds, but a variety of diagnostic stool assays of viral are available, including immune-based assays (eg, enzyme-linked immunosorbent assay) and nucleic acid testing gastroenteritis (eg, polymerase chain reaction [PCR]).2,7 made?
What are the causes of noninvasive viral diarrhea?
This virus is Norovirus. commonly associated with outbreaks of gastroenteritis on cruise ships. The leading cause of Rotavirus.2 diarrheal illness in infants and toddlers. More often Adenovirus. associated with respiratory tract infection. Named after the Astrovirus.8 Greek word for “star” because of its characteristic appearance on electron microscopy.
What are the clinical features of norovirus gastroenteritis?
Norovirus is the predominant cause of acute infectious diarrhea in adults and older children, and is often responsible for outbreaks in community settings such as recreational camps, cruise ships, and nursing homes. As a result of the large burden of organisms in stool and emesis, and the low inoculum required to cause infection, it is highly contagious. Symptoms begin 12 to 48 hours after exposure. Manifestations include fever, myalgias, headache, vomiting, and voluminous watery diarrhea. The duration of illness is typically 1 to 2 days.2,7
What are the clinical features of rotavirus gastroenteritis?
What are the clinical features of enteric adenovirus gastroenteritis? What are the clinical features of astrovirus gastroenteritis?
Rotavirus is the predominant cause of acute infectious diarrhea in young children but also affects adults. As a result of its ability to survive on environmental surfaces and the low inoculum required for infection, it is highly contagious. When a member of a family becomes infected, around one-half of children and up to one-third of adults in the household will become infected. In temperate climates, infection rates are highest during the winter months. Symptoms begin 1 to 3 days after exposure. Manifestations include fever, vomiting, and voluminous watery diarrhea, frequently requiring hospitalization for rehydration. However, most adults are asymptomatic. The duration of illness is typically 5 to 7 days but may be longer in immunocompromised hosts.2,7 Enteric adenoviruses are a common cause of acute infectious diarrhea in infants and young children. Most infected adults are asymptomatic. Symptoms begin 8 to 10 days after exposure, and the duration of illness is typically 5 to 12 days, which is a longer incubation period and duration of illness than other viral causes of diarrhea. Manifestations include fever, vomiting, and prolonged watery diarrhea. Unlike conventional adenovirus serotypes, enteric adenoviruses do not cause nasopharyngitis and keratoconjunctivitis.2,7 Astrovirus gastroenteritis primarily affects infants and young children, but outbreaks do occur in adult communities (eg, military recruits, nursing homes for the elderly). Symptoms begin 1 to 2 days after exposure. Manifestations are similar to rotaviral illness but less severe, and watery diarrhea is prominent. The duration of illness is typically 2 to 5 days.2,7
315
Noninvasive Bacterial Diarrhea
How is the diagnosis of noninvasive bacterial Gastrointestinal infection made?
The diagnosis of most noninvasive bacterial GI infections is made clinically, but culture isolation from stool is confirmatory; culture-independent techniques (eg, PCR assays to detect organism-specific toxins) may be useful in some cases.
What are the causes of noninvasive bacterial diarrhea?
Antibiotic use commonly precedes infection with this organism. A spore-forming gram-positive bacterium that causes foodborne illness, commonly involving meat and poultry. This organism, commonly found on skin, causes foodborne illness via preformed toxins. Fried rice is the classic source of this bacterium in foodborne illness. The most common cause of GI infection among travelers. Another noninvasive strain of Escherichia coli. A gram-positive, rod-shaped organism that most often affects patients with impaired cell-mediated immunity. An organism that causes severe secretory diarrhea. A gram-positive, non-acid-fast, periodic acid-Schiff (PAS)– positive bacillus.
Clostridium difficile.
Clostridium perfringens.
Staphylococcus aureus.
Bacillus cereus.
Enterotoxigenic Escherichia coli (ETEC).9
Enteropathogenic Escherichia coli (EPEC).
Listeria monocytogenes.
Vibrio cholerae.
Tropheryma whipplei.
316
What are the Clostridium difficile colitis follows the disruption of normal intestinal bacterial flora, usually as a result of antibiotic use. clinical features Manifestations of Clostridium difficile colitis vary widely, from self-limited diarrhea to life-threatening bowel of Clostridium perforation. Severe disease is defined by the presence of leukocytosis ≥15 K/µL or serum creatinine >1.5 mg/dL. difficile colitis? Fulminant disease refers to the presence of hypotension or shock, ileus, or megacolon. Additional manifestations of fulminant disease may include delirium, abdominal distention, marked leukocytosis (>35 K/µL), lactic acidosis, and evidence of end-organ failure. Oral vancomycin or oral fidaxomicin is the pharmacologic treatment of choice for non-severe infection, whereas oral vancomycin alone or in combination with parenteral metronidazole should be used for more severe cases. The infection is easily spread from person to person, and contact and isolation precautions should be taken.10,11 What are the Clostridium perfringens is a common cause of foodborne illness. Toxin-mediated symptoms typically appear 8 to clinical features 12 hours after ingestion of contaminated food and include intense abdominal cramps and watery diarrhea. The of Clostridium duration of illness is typically <24 hours but can be protracted in the elderly.12 perfringens foodborne illness? What is the Symptoms of staphylococcal foodborne illness can begin as early as one-half hour after ingestion of contaminated food timing of because of the presence of preformed enterotoxins (no incubation period is necessary). Nausea, vomiting, and symptom onset abdominal cramps usually occur first, sometimes followed by fever and diarrhea.13 after ingestion of food contaminated with Staphylococcus aureus? What are the Bacillus cereus foodborne illness is associated with 2 distinct clinical presentations. The first is marked by vomiting clinical features within 30 minutes to a few hours after ingesting contaminated food, sometimes followed by abdominal cramps and of Bacillus diarrhea, with resolution of symptoms in less than 24 hours (similar to staphylococcal food poisoning). The second, cereus foodborne referred to as diarrhea-type illness, is marked by watery diarrhea and abdominal cramps 6 to 15 hours after ingesting illness? contaminated food, lasting around 24 hours (similar to Clostridium perfringens foodborne illness).12 What are the ETEC GI illness occurs after ingestion of contaminated food or water and is characterized by rapid onset watery clinical diarrhea with or without abdominal pain, malaise, nausea, and vomiting. It typically occurs in travelers to developing manifestations of countries, with a peak onset on the second or third day after arrival. The duration of illness is typically 1 to 3 days. A enterotoxigenic short course (1-3 days) of antibiotics can decrease the duration of illness and may be appropriate in some cases.9,14 Escherichia coli Gastrointestinal illness? What Diarrheal illness caused by EPEC predominantly occurs in infants, particularly in developing countries.9 demographic is at highest risk for enteropathogenic Escherichia coli Gastrointestinal illness? What are the Listeria monocytogenes is a rare cause of foodborne gastroenteritis in healthy persons, characterized by fever, watery clinical features diarrhea, arthromyalgias, and headache. Symptoms begin ≤24 hours after exposure, and the duration of illness is of Listeria typically 1 to 3 days. It more commonly develops in patients with impaired cell-mediated immunity, such as neonates, monocytogenes pregnant women, elderly persons, and patients on chronic immunosuppressive medication, where it is frequently Gastrointestinal associated with bacteremia and meningoencephalitis.15 illness? What are the Vibrio cholerae causes toxin-mediated GI disease after ingestion of contaminated water. It is primarily a problem in clinical features developing countries with poor sanitation, where it is usually associated with seasonal outbreaks. Infection results in of Vibrio severe secretory diarrhea with a high rate of mortality, particularly in those who are untreated. Aggressive fluid cholerae resuscitation is the cornerstone of management.16 Gastrointestinal illness? What are the Tropheryma whipplei is a ubiquitous organism and the causative agent of Whipple disease, a chronic illness that clinical features develops in a minority of exposed patients. Although it has been described in patients of all ages throughout the of Whipple world, there is a predilection for middle-aged white men. It often affects multiple systems, producing a variety of disease? clinical features. The manifestations of “classic” Whipple disease are intermittent migratory arthralgias or arthritis, diarrhea, abdominal pain, and weight loss. Neurologic and cardiac involvement can occur as part of classic Whipple disease, or independently without the classic clinical picture. Diagnosis is usually made with small bowel biopsy demonstrating PAS-positive macrophages within the lamina propria. Without treatment with antibiotics, the disease is fatal. Even with treatment, there is a significant chance of recurrence.17
317
318
Noninvasive Protozoal Diarrhea
How is the diagnosis of protozoal Gastrointestinal infection made?
Protozoal GI infections are typically diagnosed via microscopic examination of stool specimens for the presence of ova and parasites. A variety of other methods (eg, PCR) are also available.
What are the causes of noninvasive protozoal diarrhea?
A flagellated organism first identified in the stool of its discoverer in the 17th century. These 2 protozoa most commonly affect immunocompromised patients. This endemic protozoan mostly affects local populations and travelers.
What are the clinical features of giardiasis?
What complication should be considered when a patient with human immunodeficiency virus (HIV) develops cholestatic liver injury during the course of cryptosporidiosis? What are the clinical manifestations of Cystoisospora Gastrointestinal infection? What are the clinical features of Cyclospora Gastrointestinal infection?
Giardia lamblia (ie, Giardia duodenalis or Giardia intestinalis).18
Cryptosporidium species and Cystoisospora belli.
Cyclospora cayetanensis.
Giardiasis is a common cause of diarrhea across the world, acquired by ingesting cysts from contaminated water or food. It affects patients of all ages, with a peak incidence in the late summer and fall. Symptoms begin 7 to 14 days after exposure. Manifestations include diarrhea, malaise, flatulence, foul-smelling and greasy stools, abdominal cramps, bloating, nausea, anorexia, and weight loss. The duration of illness is prolonged, often
14 days. A variety of diagnostic stool assays are available. Treatment options include metronidazole, tinidazole, or nitazoxanide.18 Cryptosporidiosis in patients with HIV infection or acquired immunodeficiency syndrome (AIDS) is typically characterized by chronic diarrhea, foul-smelling and bulky stools, and weight loss. Biliary involvement (eg, sclerosing cholangitis) can occur, particularly when CD4 cell counts are <50/µL, and should be suspected when serum alkaline phosphatase levels become elevated during the course of illness. Suppressing viremia and preventing loss of CD4 cells are the cornerstone of managing cryptosporidiosis in patients with HIV. The antiparasitic agent nitazoxanide may also be beneficial. Procedural intervention may be necessary for biliary complications. In immunocompetent hosts, cryptosporidiosis presents with watery diarrhea with or without abdominal cramps, nausea, vomiting, and fever. The illness is self-limited with a typical duration of 5 to 10 days.18
Cystoisospora most commonly infects immunocompromised patients. Its clinical course is indistinguishable from that of Cryptosporidium. However, unlike Cryptosporidium, highly effective treatment is available; patients respond promptly to trimethoprim-sulfamethoxazole.19
Manifestations of Cyclospora infection include low-grade fever, abdominal cramps, diarrhea, anorexia, nausea, flatulence, fatigue, and weight loss. In endemic areas, children and the elderly tend to have more severe presentations, while asymptomatic infections are common in nonelderly adults. In travelers or in individuals affected by outbreaks in nonendemic areas, Cyclospora infection is invariably symptomatic and tends to be more severe in HIV or AIDS patients. Symptoms begin around 7 days after exposure, and untreated illness often lasts weeks to months, although is even longer in HIV or AIDS patients. Trimethoprim-sulfamethoxazole is the treatment of choice in any population with symptomatic infection, significantly shortening the duration of illness.20
319
320
Invasive Infectious Diarrhea
What clinical features are suggestive of invasive infectious diarrhea? How is the diagnosis of invasive bacterial Gastrointestinal infection made?
Invasive infectious diarrhea often presents with abdominal pain, fever, tenesmus, and bloody or mucoid stools. It is frequently associated with extraintestinal manifestations such as inflammatory arthritis and conjunctivitis.1,3 The diagnosis of invasive bacterial GI infection is typically made via culture isolation from stool; culture-independent techniques (eg, PCR) may be useful in some cases.
What are the causes of invasive infectious diarrhea?
Typhoidal and nontyphoidal subtypes. A 38-year-old man develops ascending paralysis following a bout of gastroenteritis. Invasive strains of Escherichia coli. Discovered at the end of the 19th century in Japan during an investigation of an outbreak of “sekiri” (meaning “red diarrhea”). A 43-year-old woman develops diarrhea after consuming raw pork while vacationing in Hawaii. A protozoan often associated with liver abscesses. A previously healthy patient develops abdominal pain and diarrhea 1 day after consuming raw shellfish. Most symptomatic and invasive infections from this virus occur in immunocompromised hosts.
What are the clinical features of Salmonella Gastrointestinal infection?
Salmonella species.
Campylobacter species.
Enterohemorrhagic Escherichia coli (EHEC) and enteroinvasive Escherichia coli (EIEC). Shigella species.21
Yersinia species.
Entamoeba histolytica.
Vibrio parahaemolyticus.
Cytomegalovirus (CMV).
Salmonella species are capable of causing 2 different GI presentations: gastroenteritis and enteric fever (ie, typhoid fever or paratyphoid fever). Gastroenteritis is caused by nontyphoidal Salmonella and is one of the most common foodborne illnesses in the world. Manifestations include watery or bloody diarrhea, abdominal pain, nausea and vomiting, headache, and myalgias. Severity is greater in children, elderly, and immunocompromised patients. It is typically self-limited and lasts ≤10 days. Enteric fever collectively refers to typhoid fever (caused by Salmonella enterica serovar Typhi) and paratyphoid fever (caused by Salmonella enterica serovar Paratyphi), which are clinically indistinguishable. Enteric fever is transmitted via ingestion of contaminated food or water. It is
321
What are the clinical features of Campylobacter gastroenteritis?
What serious extraintestinal complication can occur in patients with enterohemorrhagic Escherichia coli Gastrointestinal infection? What are the clinical features of Shigella gastroenteritis?
What are the clinical features of Yersinia gastroenteritis?
What are the clinical features of Entamoeba histolytica Gastrointestinal infection?
What are the clinical manifestations of Vibrio parahaemolyticus gastroenteritis? What are the clinical features of cytomegalovirus infection of the Gastrointestinal tract?
uncommon in the industrialized world. Manifestations include watery or bloody diarrhea (sometimes constipation occurs instead of diarrhea), abdominal pain, headache, and fever. Complications include bradycardia, hepatomegaly, splenomegaly, pancreatitis, hepatitis, cholecystitis, and GI hemorrhage from perforation of Peyer’s patches. The duration of illness is weeks to months. Antibiotics are highly effective; however, multidrug resistant strains of serovars Typhi and Paratyphi are on the rise. Salmonella enterica 22 Campylobacter gastroenteritis most often develops after ingestion of contaminated food, particularly poultry meat. Infection is common in the developing world; it occurs as outbreaks in the industrialized world. Symptoms begin 1 to 3 days after exposure. Manifestations include watery or bloody diarrhea, fever, weight loss, and abdominal cramps. It is self-limited with an average duration of 6 days. Campylobacter gastroenteritis can be associated with various extraintestinal manifestations, including Guillain-Barré syndrome.23 EHEC infection is most often caused by Escherichia coli O157:H7, a Shiga toxin–producing invasive organism usually acquired via ingestion of contaminated food or water. Symptoms typically begin 3 days after ingestion but may present up to 12 days later. Early manifestations (within the first 1 to 3 days) include severe abdominal cramping, vomiting, watery diarrhea, and fever. After the initial phase, the diarrhea typically becomes bloody. The most serious complication of EHEC infection is hemolytic uremic syndrome (HUS), which is characterized by the triad of hemolytic anemia, thrombocytopenia, and Acute Kidney Injury. It occurs in up to one-fifth of patients with EHEC infection. Use of antibiotics does not shorten the duration of illness related to EHEC infection, and there is some evidence that suggests it is associated with the development of HUS.24 Shigellosis is most commonly caused by Shiga toxin–producing Shigella dysenteriae, transmitted via the fecal-oral route, usually as a result of person-to-person contact or ingestion of contaminated food or water. The vast majority of cases occur in developing countries. In the United States, individuals at highest risk include children in day-care centers, migrant workers, travelers to developing countries, and homosexual men. Symptoms begin 1 to 4 days after exposure, but may present up to 8 days later. Early manifestations (within the first 1 to 2 days) include fever, fatigue, malaise, anorexia, and watery diarrhea. Abdominal cramps, tenesmus, and mucoid and bloody stools follow the initial phase. Intestinal complications (eg, toxic megacolon) and extraintestinal complications (eg, HUS, seizure, reactive arthritis) can occur. In previously healthy individuals, the illness is self- limited, lasting 5 to 7 days. EIEC illness is closely related to Shigella with indistinguishable clinical manifestations.21,25 Yersiniosis is most commonly caused by Yersinia enterocolitica, usually acquired via ingestion of contaminated food, particularly undercooked or raw pork. Infections occur worldwide but are most common in European countries, with a predilection for the winter months. Symptoms begin 1 to 11 days after exposure. Typical manifestations include fever, abdominal pain, and watery or bloody diarrhea; nausea, vomiting, and pharyngitis sometimes occur. It can also present with a pseudoappendicular syndrome (a mimic of acute appendicitis), which is more common in children and young adults. Postinfectious complications are more common in adults (particularly Scandinavian women), and include reactive arthritis and erythema nodosum. In uncomplicated cases, yersiniosis is self-limited and typically lasts 5 to 14 days.26 Intestinal amebiasis develops after ingestion of contaminated food or water. It occurs worldwide but is more common in developing countries. Infection is often asymptomatic. Clinical manifestations are insidious in onset, typically occurring over weeks, and include cramping abdominal pain, weight loss, and watery or bloody diarrhea. Severe manifestations include necrotizing colitis, toxic megacolon, ameboma (a mass of colonic granulation tissue), and perianal ulceration. Metronidazole is the treatment of choice. Amebic liver abscess is the most common extraintestinal manifestation; it is significantly more common in men than women or children.27 Vibrio parahaemolyticus lives in marine or estuarine environments, typically causing gastroenteritis when contaminated seafood is ingested. It is a worldwide disease; in the United States, it occurs in foodborne outbreaks. Symptoms typically begin 17 hours after exposure (range 4-90 hours). Watery or bloody diarrhea is the most frequent symptom and may be accompanied by abdominal cramps, nausea, and vomiting. The illness is self- limited, lasting on average 2.5 days (range 8 hours to 12 days).28 CMV infection of the GI tract most often affects immunocompromised patients such as those with HIV infection or AIDS and those on chronic immunosuppressive medication. Although clinical manifestations are more severe in immunocompromised individuals, it does occur in immunocompetent hosts. CMV can affect any part of the GI tract, but the colon is most commonly involved. Manifestations include abdominal pain, fever, weight loss, and watery or bloody diarrhea. Ulcerative and erosive lesions are frequently observed on endoscopic evaluation; biopsy is necessary to confirm the diagnosis. CMV colitis cannot be excluded based on a negative plasma or whole blood PCR test. Patients with symptomatic CMV GI disease should be treated with antiviral therapy.29,30
322
Noninfectious Inflammatory Diarrhea
What are the causes of noninfectious inflammatory diarrhea?
Look for extraintestinal Inflammatory bowel disease. manifestations such as erythema nodosum (Figure 15- 3).
FIGURE 15-3 Erythema nodosum on the lower extremity of a patient with an acute flare of inflammatory bowel disease.
An 80-year-old woman presents with mild left lower quadrant abdominal pain and bloody diarrhea, and is found to have a blood pressure of 88/55 mm Hg. Streptococcus bovis endocarditis. Look for tattoo marks around the abdominal area. A previously healthy 36-year-old woman presents with several months of nausea and vomiting, diarrhea, and painful neuropathy; her husband recently Increased her life insurance policy.
What must be ruled out in any patient with a history of inflammatory bowel disease who presents with a flare? What findings on cross- sectional imaging can help distinguish ischemic colitis from other causes of colitis, including infection and
Ischemic colitis.
Colorectal cancer (CRC). External beam radiation therapy. Arsenic poisoning.
When patients with IBD present with acute symptoms, infectious enterocolitis must be ruled out with stool evaluation. Enteric infection can be found in 10% of these patients, and the most common agent is Clostridium difficile.31 Computed tomography (CT) imaging can be a valuable tool in working up patients with diarrhea, particularly when it is bloody. Intestinal wall thickening and fat stranding on CT imaging are common features of enterocolitis, regardless of underlying etiology. Ischemic colitis is suggested when these changes occur in a vascular or “watershed” distribution (eg, distal transverse colon and distal descending colon) (see Figure 18-3).32
323
inflammatory bowel disease? What are the risk factors for colorectal cancer?
How long after external beam radiation therapy do patients develop radiation enteropathy?
What are the characteristics of diarrhea associated with arsenic toxicity?
CRC is one of the most common cancers in the world. The lifetime risk of CRC in the average American is 1 in 20. Nonmodifiable risk factors include age, family history of CRC, and IBD. Modifiable risk factors include alcohol use, obesity, smoking, and consuming processed and red meats.33 Radiation enteropathy is a common side effect of external beam radiation therapy used to treat abdominal and pelvic cancers. Incidence is directly related to the dose of radiation received. Most cases are acute, occurring at the time of or shortly after therapy (within 90 days postradiation), and are generally reversible. However, delayed-onset radiation enteropathy can develop beyond 90 days postradiation (up to decades later) and is typically not reversible.34 Acute arsenic toxicity results in voluminous diarrhea similar to that caused by cholera, except it is bloody. It is described as “bloody rice water” diarrhea. It can result in death from hypovolemic shock. The diarrhea of chronic arsenic toxicity is intermittent and may be associated with vomiting. Arsenic poisoning should be suspected when other characteristic extraintestinal manifestations are present, including dermatologic manifestations (eg, diffuse hyperpigmentation, palmar keratosis) and neurologic manifestations (eg, peripheral neuropathy). Hair sample analysis can reveal the timeline of arsenic exposure.35
324
Osmotic Diarrhea
What is the fundamental mechanism of osmotic diarrhea? What historical features are suggestive of osmotic diarrhea? What stool study is suggestive of osmotic diarrhea?
In which setting should stool osmolality be directly measured (rather than assumed to be 290 mOsm/kg)? Which test is used to investigate osmotic diarrhea related to malabsorption?
Osmotically active particles can accumulate in the intestinal lumen as a result of ingestion of nonabsorbable substances, maldigestion, or malabsorption, generating an osmotic gradient that favors the movement of fluid into the lumen, resulting in diarrhea.3
The volume of osmotic diarrhea decreases during periods of fasting, including at night. Weight loss and steatorrhea are clues to the presence of maldigestion or malabsorption. Steatorrhea describes greasy, bulky, and malodorous stools that are difficult to flush, often leaving an oil residue on the toilet bowl.3,36
Osmotic diarrhea is associated with an elevated stool osmotic gap (the difference between measured and calculated osmolality of stool fluid). Stool osmolality can be directly measured, or assumed to be 290 mOsm/kg (similar to serum osmolality). Because stool fluid is normally electroneutral, osmolality is calculated by multiplying the sum of sodium and potassium concentrations of stool fluid by a factor of 2 (to account for the anions). Stool osmotic gap =measured osmolality −(2 ×[Na +K]). An osmotic gap >50 mOsm/kg suggests the presence of osmotically active substances in stool fluid. An osmotic gap <50 mOsm/kg suggests a secretory process.3,37,38 When factitious diarrhea is suspected, and there is a possibility that the stool sample has been surreptitiously diluted, stool osmolality should be directly measured. Stool osmolality that is unexpectedly high (eg, >600 mOsm/kg) suggests that the sample was likely an admixture of stool and urine.5,39
Small intestine malabsorption is suggested by a positive d-xylose absorption test. The patient is given a dose of d-xylose, and its concentrations in serum and urine are subsequently measured. If serum and urine levels of d-xylose are abnormally low, it suggests small intestine malabsorption. False positive results can occur in patients with renal dysfunction and small bowel bacterial overgrowth. Because pancreatic enzymes are not necessary for the absorption of xylose, this test is unreliable for the detection of malabsorption caused by pancreatic insufficiency.40
What are the causes of osmotic diarrhea?
A patient complains of recurrent diarrhea within a few hours of consuming dairy products. Used to treat constipation but sometimes abused. A 23-year-old man complains of watery diarrhea after he eats his favorite sugar- free candy. The small intestine normally contains fewer bacteria than the large intestine. Villous atrophy is the hallmark histologic finding of this autoimmune condition. An extrapulmonary sequela of cystic fibrosis. Look for surgical scars. Impaired enterohepatic circulation.
Lactose intolerance.
Laxatives.
Ingestion of a sugar alcohol (eg, sorbitol).
Small intestinal bacterial overgrowth.
Celiac disease.
Pancreatic exocrine insufficiency.
Short bowel syndrome. Cholerheic diarrhea.
325
A disease of the Tropical sprue. tropics, affecting both indigenous populations and travelers who stay for more than a few weeks.
How is lactose Digestion of dietary lactose requires the enzyme lactase-phlorizin hydrolase (ie, lactase), which is present on the apical intolerance surface of enterocytes of the small intestine. Lactase hydrolyzes lactose to produce glucose and galactose, which can diagnosed? then be absorbed by intestinal enterocytes. The activity of lactase declines in most individuals after infancy, resulting in lactose malabsorption, but only some will develop symptoms (ie, lactose intolerance). Lactose intolerance occurs when undigested lactose is fermented in the colon, producing symptoms such as abdominal pain, bloating, flatulence, diarrhea, and borborygmi. A presumptive diagnosis can be made in patients with a compatible clinical history. The lactose hydrogen breath test detects the production of hydrogen from lactose fermentation and can confirm the diagnosis in most patients.41 What Medications that cause osmotic diarrhea include antibiotics (eg, ampicillin, clindamycin), enteral feeding formulas, medications can laxatives (eg, magnesium salts, polyethylene glycol, lactulose), prebiotics, methyldopa, quinidine, propranolol, cause osmotic hydralazine, angiotensin-converting enzyme inhibitors, and procainamide.5 diarrhea? What sugar Sugar alcohols are often used as artificial sweeteners, including sorbitol, mannitol, xylitol, erythritol, lactitol, maltitol, alcohols are and glycerol. Sorbitol-containing excipients are included in many pharmaceutical agents. Diarrhea can develop by commonly used consuming as little as 10 g/d of sorbitol.5 as artificial sweeteners? How is small Small intestinal bacterial overgrowth is associated with a number of underlying conditions, including anatomic intestinal abnormalities (eg, small intestinal diverticulosis, strictures in patients with Crohn’s disease, surgery) and abnormal bacterial small intestinal motility (eg, diabetes, scleroderma, hypothyroidism, amyloidosis, radiation enteritis). The principles of overgrowth management include treatment of the underlying condition (if possible), eradication of overgrowth with antibiotics, and managed? correction of associated nutritional deficiencies.42 What are the The most common symptoms of celiac disease are chronic diarrhea, weight loss, and abdominal distention. Other clinical features manifestations include iron deficiency, abdominal pain, aphthous stomatitis, aminotransferase elevation, and chronic of celiac fatigue. Patients who do not have IgA deficiency (which is associated with celiac disease) should be tested for the disease? presence of serum IgA anti–tissue transglutaminase, which is associated with high sensitivity and specificity. False positives may occur in patients with other autoimmune conditions (eg, type 1 diabetes mellitus). In patients with IgA deficiency, IgG anti–tissue transglutaminase antibodies should be measured instead. Measurement of IgA anti-endomysial antibodies may be helpful in patients with borderline anti–tissue transglutaminase levels or in those who might have false positive results. The diagnosis is definitively established with biopsy of the small intestine.43 What are the Pancreatic exocrine insufficiency most commonly occurs as a result of pancreatic parenchymal disease (eg, chronic causes of pancreatitis, cystic fibrosis, necrotizing acute pancreatitis, pancreatic resection), pancreatic duct obstruction (eg, tumor, pancreatic stricture), or inadequate stimulation of pancreatic enzyme secretion (eg, small bowel resection, celiac disease). The gold exocrine standard diagnostic test is 72-hour fecal fat quantification; however, this test is inconvenient. Fecal elastase-1 insufficiency? measurement can be used to make the diagnosis instead. When results are equivocal but clinical suspicion persists, direct tests of pancreatic exocrine function can be performed (eg, pancreatic stimulation with secretin or cerulein).44 What is short Short bowel syndrome describes malabsorption that occurs after small intestine resection. The normal length of small bowel intestine varies from 275 to 850 cm and is generally longer in men. Malabsorption typically develops when <200 cm of syndrome? small intestine remains. Common reasons for short bowel syndrome include Crohn’s disease, superior mesenteric artery thrombosis, and radiation enteritis. It is more common in women by a ratio of 2:1 likely because, compared to men, women have a shorter length of small intestine.45 What is Bile acids produced in the liver and secreted into the small intestine are normally reabsorbed in the terminal ileum for cholerheic reuse (ie, enterohepatic circulation). Cholerheic diarrhea occurs when bile acids are not effectively reabsorbed in the diarrhea? ileum and reach the colon, where their presence leads to diarrhea by stimulating colonic electrolyte and water secretion and increasing colonic motility. Conditions associated with bile acid malabsorption include cholecystectomy, ileal inflammation or resection (usually related to Crohn’s disease), small intestinal bacterial overgrowth, and pancreatic insufficiency. Bile acid sequestrants, such as cholestyramine, are an effective treatment option for most patients.46 What type of Tropical sprue is an acquired malabsorptive syndrome endemic to certain regions of the world, including parts of Asia, anemia is found some Caribbean islands, and parts of Latin America, affecting indigenous populations and travelers. It is thought to be in patients with infectious in nature. Patients develop chronic diarrhea, anorexia, weight loss, and megaloblastic anemia from vitamin tropical sprue? B12 or folate deficiency, which can be an important clue to the diagnosis. Treatment with tetracycline is highly effective.47
326
327
Secretory Diarrhea
What is the fundamental mechanism of secretory diarrhea? What historical features can be suggestive of secretory diarrhea? What stool study is suggestive of secretory diarrhea?
Secretory diarrhea occurs as a result of the secretion of excess isotonic fluid into the intestine.3
The volume of secretory diarrhea, which is typically high, does not change during periods of fasting, including at night. Watery diarrhea is typical; the presence of steatorrhea or blood is unusual.3 A stool osmotic gap <50 mOsm/kg is suggestive of secretory diarrhea.3,37,38
What are the causes of secretory diarrhea?
A patient develops diarrhea after starting treatment for an acute gout flare. A neuroendocrine tumor that secretes serotonin. A neuroendocrine tumor that secretes gastrin. A neuroendocrine tumor that secretes glucagon. A neuroendocrine tumor that secretes vasoactive intestinal peptide (VIP). Excess histamine.
What medications can cause secretory diarrhea? What is carcinoid syndrome? What are the clinical features of gastrinoma? What are the clinical features of glucagonoma?
Colchicine.
Carcinoid.
Gastrinoma.
Glucagonoma.
VIPoma.
Systemic mastocytosis.
Numerous medications can cause secretory diarrhea, some of which include antibiotics (eg, amoxicillin-clavulanate), laxatives (eg, senna glycoside, bisacodyl), caffeine, digoxin, carbamazepine, calcitonin, cimetidine, chemotherapeutic agents (eg, idarubicin), and metformin.5 Carcinoid syndrome occurs when serotonin and other vasoactive substances secreted by neuroendocrine tumors reach the systemic circulation, resulting in characteristic clinical manifestations such as abdominal pain, diarrhea, and episodic facial flushing.48 Gastrinomas cause gastric acid hypersecretion, which leads to abdominal pain, gastroesophageal reflux disease, peptic ulcer disease, and chronic diarrhea. Around one-half of gastrinomas are malignant. Diagnostic laboratory tests include fasting serum gastrin level and the secretin stimulation test.49 Glucagonomas are associated with glucose intolerance and a skin rash characterized by painful and pruritic erythematous plaques that are well-demarcated and often located in intertriginous areas (necrolytic migratory erythema). Chronic diarrhea is present in some patients. Most glucagonomas are malignant. Elevated plasma glucagon level >1000 pg/mL is diagnostic.49
328
What are the clinical features of VIPoma? How common are Gastrointestinal symptoms in patients with systemic mastocytosis?
VIPomas are associated with intermittent, severe, watery diarrhea, sometimes in excess of 5 L/d, which leads to hypokalemia and metabolic acidosis in most patients. Most VIPomas are malignant. Elevated plasma VIP level is diagnostic. However, VIP secretion is often episodic and may be missed with a single measurement. 49 Mastocytosis refers to the proliferation of mast cells within the skin (cutaneous mastocytosis) or extracutaneous tissues (systemic mastocytosis) with or without skin involvement. The vast majority of patients with systemic mastocytosis experience GI symptoms, the most common of which are abdominal pain and diarrhea. In patients with mastocytosis, the release of mast cell mediators can be precipitated by various stimuli, including medication (eg, nonsteroidal anti-inflammatory drugs), alcohol, and stress. Avoiding triggers is a critical part of management; a variety of pharmacologic agents can also be used to treat the GI symptoms of mastocytosis, including antihistamines, cromolyn, antileukotriene drugs (eg, montelukast), and budesonide.50
329
Diarrhea Related to Intestinal Dysmotility
What are the mechanisms of diarrhea related to intestinal dysmotility? What historical features can be suggestive of diarrhea related to dysmotility?
Hypermotility decreases transit time of intestinal contents, preventing reabsorption of fluid in the small intestine and overwhelming the absorptive capacity of the large intestine. Hypomotility is associated with the development of small intestinal bacterial overgrowth.3 Intestinal dysmotility often produces small-volume, high-frequency stools. Steatorrhea is common, although the presence of blood suggests an alternative mechanism. Dysmotility is often a symptom of a systemic disease (eg, diabetes mellitus) that can frequently be recognized by the presence of characteristic extraintestinal manifestations.3
What are the causes of diarrhea related to intestinal dysmotility?
One of the most common GI conditions, present in up to one-fifth of the general population. A 34-year-old woman develops diarrhea after starting treatment for myasthenia gravis. Gastroparesis is the more typical GI manifestation of this common systemic disease. A 36-year-old woman with diarrhea, weight loss, rapid heart rate, and fine tremor. A 48-year-old woman with sclerodactyly, Raynaud’s phenomenon, and dysphagia. A 56-year-old man with weight loss, autonomic dysfunction, nephrotic syndrome, and elevated serum protein gap.
What is irritable bowel syndrome?
Irritable bowel syndrome (IBS).51
Acetylcholinesterase inhibitor.
Diabetes mellitus.
Thyrotoxicosis.
Scleroderma (ie, systemic sclerosis).
Amyloidosis.
IBS is a symptom-based condition characterized by abdominal pain and altered bowel habits, including constipation, diarrhea, or both. It is a chronic condition in most patients, but symptoms may vary over time. IBS is somewhat of a diagnosis of exclusion; concerning clinical features should always be investigated to rule out more harmful
330
What medications can cause diarrhea related to dysmotility? What are the clinical features of the diarrhea caused by diabetes mellitus?
How common is diarrhea in patients with thyrotoxicosis? How common is diarrhea in patients with systemic sclerosis? How common is diarrhea in patients with amyloidosis?
conditions. Concerning features include the onset of symptoms after 50 years of age, unexplained weight loss, family history of organic GI disease (eg, colon cancer, IBD), GI bleeding, or unexplained iron-deficiency anemia.51 Medications that can cause diarrhea related to dysmotility include acetylcholinesterase inhibitors (eg, pyridostigmine), cholinergics (eg, bethanechol), prokinetic agents (eg, metoclopramide, cisapride), macrolides (eg, erythromycin), and colchicine.5 Diabetes mellitus more often presents with symptoms and signs of hypomotility, such as gastroparesis. However, diarrhea occurs in up to one-fifth of patients with diabetes as a result of dysmotility, small intestinal bacterial overgrowth, and pancreatic exocrine insufficiency. It is watery or fatty in nature and tends to be intermittent, often separated by periods of normal bowel movements or constipation. It most often occurs in middle-aged diabetics with long-standing and poorly-controlled disease; most patients have concurrent peripheral neuropathy and evidence of autonomic dysfunction elsewhere (eg, bladder dysfunction). Treatment is often unsatisfactory.52 Up to one-quarter of patients with thyrotoxicosis experience diarrhea, which is usually small-volume but high-frequency. Steatorrhea is present in some. Patients with Graves’ disease have an Increased risk of concurrent celiac disease. In addition to addressing the underlying disease, β-antagonists (eg, propranolol) can be effective in treating the diarrhea of thyrotoxicosis.53 The vast majority of patients with systemic sclerosis have GI involvement. Symptoms and signs of hypomotility such as constipation are common. Diarrhea is also common and is mostly the result of small intestinal bacterial overgrowth related to intestinal stasis. Patients often complain of alternating episodes of constipation and diarrhea. Because of anorectal involvement, fecal incontinence is also common in these patients.54 Amyloidosis more often presents with symptoms and signs of hypomotility. However, chronic diarrhea occurs in up to 15% of patients with amyloidosis, as a result of dysmotility, intestinal inflammation, and small intestinal bacterial overgrowth. It can be treated with loperamide or octreotide. Intractable diarrhea is associated with poor survival; less than one-half of patients are alive after 5 years.55
331 Case Summary diarrhea, weight loss, facial flushing, and dyspneaiand is found toon A 58-year-old man presents with chronic volum nous watery have a murmur on examination and numerous lesions of the liver cross-sectional imaging.
What is the most likely cause of diarrhea in this patient? Carcinoid syndrome.
332
Bonus Questions
What features Persistent voluminous watery diarrhea despite fasting is characteristic of secretory diarrhea. The stool osmotic gap <50 mOsm/kg in this case is also consistent with the consistent with in this case are diagnosis. secretory diarrhea? Where are most Carcinoid tumors describe well-differentiated neuroendocrine tumors. Most are located intthe GI tract (particularly the small intestine) or lung. Carcinoid syndrome in the body? tumors located carcinoid occurs in up to one-half of patients with carcinoid tumors of the GI tract but in only 5% of hose with bronchial tumors. 48 Why does When carcinoid tumors are limited to the GI tract, vasoactive substances are secreted into the portal circulation and subsequently inactivated by the liver, preventing carcinoid carcinoid syndrome from developing. When there is hepatic involvement, vasoactive substances are secreted directly into the systemic circulation, which is necessary syndrome for the syndrome to develop. 48 suggest the presence of liver
metastases? What are the Flushing is the most common manifestation of carcinoid syndrome, occurring in the vast majority of patients. It is intermittent, sudden in onset, typically involves the features of chocolate, walnut,lbanana, pineapple, tomato, plum, avocado). Around one-half of patients experience abdominal pain,iparticularly after large meals. Other significance of distorted, leading to stenosis, regurgitation, or both. The left-sided valves are spared in the vast majority oftcases because the lungs inactivate the vasoactive substances clinical face and upper trunk, and lasts 5 to 10 minutes. Diarrhea occurs in most patients. Flushing and diarrhea can be triggered by stress, alcohol, and certain foods (eg, syndrome? carcinoid manifestations inc ude wheezing (more common in patients with bronchial tumors), weight loss, and features of right-s ded heart failure. 48 What is the The vasoactive substances released by carcinoid tumors cause endocardial fibrotic plaques, involving struc ures such as valves and subvalvular apparati, which become the heart before they reach the left side of the heart. The decrescendo diastolic murmur in this case could be consistent with either aortic or pulmonic regurgitation. However, the murmur in this presence of Carvallo’s sign (murmur intensity increases with inspiration), which is indicative of right-sided valvular lesions, suggests pulmonic regurgitation. For an case? What is the Sero onin secreted by carcinoid tumors is metabolized to 5-HIAA; ur nary excretion of 5-HIAA can be measured with a random urine test or a 24-hour urine collectio audio/video demonstration of Carvallo’s sign, see the associated reference.56,57 most useful bothtof which are associated with excellent sensitivity and specificityi(diet must be free of tyramine for >24 hours before urine collection). A positive urinary 5-HIAA n, diagnostic test scan shows areas of normal radiotracer uptake (eg, bladder, kidneys, spleen), but there is a focus of Increased uptake in the region of the small intestine (the primary initial test should be followed by measurement of serum chromogranin A and an octreotide scan, to confirm the diagnosis and stage the disease. In this case, the octreotide for carcinoid laboratory tumor site), and multiple globular foci of Increased uptake in the liver, which confirms the presence of liver metastases. 48 syndrome? How isid The symptoms of carcinoid syndrome can be abated in most patients with somatostatin analogues (eg, octreotide), or interferon alfa. Depending on the stage of disease, managed? syndrome carcino surgical tumor removal can be offered and is potentially curative. 48
333 Key Points
frequency of stools, typically occurring at leastt3 times in a 24-hour Acute diarrhea lasts ≤14 days; persistent diarrhea lasts >14 days; Diarrhea can be inflammatory, osmotic, secretory, or related to Inflammatory diarrhea can be infectious or noninfectious. Noninvasive infectious diarrhea can be viral, bacterial, or Diarrhea describes an increase in water conten , volume, or period. and chronic diarrhea lasts >30 days. intestinal dysmotility. Infectious diarrhea can be noninvasive or invasive. protozoal. Historical fea ures such as exposures (eg medications, travel history), timetcourse (eg, acute, chronic),,and stool characteristics Infection is.the most common cause of acute diarrhea; chronicity Inflammatory diarrhea caused by either invasive infections or (eg, watery, bloody, fatty) are helpful in determining the etiology of diarrhea should broaden the investigative focus. noninfectious conditions is associated with the presence of polymorphonuclear leukocytes or leukocyte proteins in stool.
Persistence of diarrhea despite fasting suggests a secretory cause. osmotic (gap >50 mOsm/kg) and secretory (gap <50 mOsm/kg) The stool osmotic gap can be helpful in distinguishing between diarrhea.
334
References 1. Guerrant RL, Van Gilder T, Steiner TS, et al. Practice guidelines for the management of infectious diarrhea. Clin Infect Dis. 2001;32(3):331-351.
-
Musher DM, Musher BL. Contagious acute Gastrointestinal infections. N Engl J Med. 2004;351(23):2417-2427.
-
Camilleri M. Chronic diarrhea: a review on pathophysiology and management for the clinical gastroenterologist. Clin Gastroenterol Hepatol. 2004;2(3):198-206.
-
Agarwal R, Afzalpurkar R, Fordtran JS. Pathophysiology;of potassium absorption and 5. Philip NA, Ahmed N, Pitchumoni CS. Spectrum of drug-induced chronic diarrhea. J secretion by the human intestine. Gastroenterology. 1994 107(2):548-571. Clin Gastroenterol. 2017;51(2):111-117.
-
Autenrieth DM, Baumgart DC. Toxic megacolon. Inflamm Bowel Dis. 2012;18(3):584- 591.
-
Blacklow NR, Greenberg HB. Viral gastroenteritis. N Engl J Med. 1991;325(4):252-264.
-
Bosch A, Pinto RM, Guix S. Human astroviruses. Clin Microbiol Rev. 2014;27(4):1048- 1074.
-
Baron S, ed. Medical Microbiology. 4th ed. Galveston, TX: University of Texas Medical Branch at Galveston; 1996.
-
Ong GK, Reidy TJ, Huk MD, Lane FR. Clostridium difficile colitis: a clinical review. Am J Surg. 2017;213(3):565-571. . McDonald LC, Gerding DN, Johnson S, et al. Clinical practice guidelines for
11Clostridium difficile infection in adults and children: 2017 update by the Infectious Diseases Society of America (IDSA) and Society for Healthcare Epidemiology of
America (SHEA). Clin Infect Dis. 2018;66(7):987-994. 12. Le.Loir Y, Baron F, Gautier M. Staphylococcus aureus and food poisoning. Genet Mol Res 2003;2(1):63-76. 13. Balaban N, Rasooly A. Staphylococcal enterotoxins. Int J Food Microbiol. 2000;61(1):1- 10.
- Barrett J, Brown M. Travellers’ diarrhoea. BMJ. 2016;353:i1937.
-
Ooi ST, Lorber B. Gastroenteritis due to Listeria monocytogenes. Clin Infect Dis. 2005;40(9):1327-1332.
-
Faruque SM, Albert MJ, Mekalanos JJ. Epidemiology, genetics, and ecology of toxigenic Vibrio cholerae. Microbiol Mol Biol Rev. 1998;62(4):1301-1314.
-
Fenollar F, Puechal X, Raoult D. Whipple’s disease. N Engl J Med. 2007;356(1):55-66. 18. Huang DB, White AC. An updated review on Cryptosporidium and Giardia. Gastroenterol Clin North Am. 2006;35(2):291-314, viii.
-
Gellin BG, Soave R. Coccidian infections in AIDS. Toxoplasmosis, cryptosporidiosis, and isosporiasis. Med Clin North Am. 1992;76(1):205-234.
-
Ortega YR, Sanchez R.iUpdate on Cyclospora cayetanensis, a food-borne and waterborne parasite. Cl n Microbiol Rev. 2010;23(1):218-234.
-
Niyogi SK. Shigellosis. J Microbiol. 2005;43(2):133-143. 22on pathogenesis, epidemiology and antibiotic resistance. Front Life Sci. 2015;8(3):284- . Eng SK, Pusparajah P, Mutalib NS, Ser HL, Chan KG, Lee LH. Salmonella: a review 293.
-
Kaakoush NO, Castano-Rodriguez N, Mitchell HM, Man SM. Global epidemiology of
335
Campylobacter infection. Clin Microbiol Rev. 2015;28(3):687-720. 24. Page AV, Liles WC. Enterohemorrhagic Escherichia coli infections and the hemolytic- uremic syndrome. Med Clin North Am. 2013;97(4):681-695, xi.
-
Clements A, Young JC, Constantinou N, Frankel G. Infection strategies of enteric pathogenic Escherichia coli. Gut Microb. 2012;3(2):71-87.
-
Cover TL, Aber RC. Yersinia enterocolitica. N Engl J Med. 1989;321(1):16-24.
-
Haque R, Huston CD, Hughes M, Houpt E, Petri WA Jr. Amebiasis. N Engl J Med. 2003;348(16):1565-1573.
-
Daniels NA,,MacKinnon L, Bishop R, et al. Vibrio parahaemolyticus infections in the 29. Chetty R, Roskell DE. Cytomegalovirus infection in the Gastrointestinal tract. J Clin United States 1973-1998. J Infect Dis. 2000;181(5):1661-1666. Pathol. 1994;47(11):968-972.
-
Klauber E, Briski LE, Khatib R. Cytomegalovirus colitis in the immunocompetent host: an overview. Scand J Infect Dis. 1998;30(6):559-564. . Mylonaki M, Langmead L, Pantes A, Johnson F, Rampton DS. Enteric infection in
31relapse of inflammatory bowel disease: importance of microbiological examination of stool. Eur J Gastroenterol Hepatol. 2004;16(8):775-778.
- Thoeni RF, Cello JP. CT imaging of colitis. Radiology. 2006;240(3):623-638.
- Johnson CM, Wei C, Ensor JE, et al. Meta-analyses of colorectal cancer risk factors. Cancer Causes Control. 2013;24(6):1207-1222. . Shadad AK, Sullivan FJ, Martin JD, Egan LJ. Gastrointestinal radiation injury:
34symptoms, risk factors and mechanisms. World J Gastroenterol. 2013;19(2):185-198. 35. Ratnaike RN. Acute and chronic arsenic toxicity. Postgrad Med J. 2003;79(933):391- 396.
-
Sweetser S. Evaluating.the patient with diarrhea: a case-based approach. Mayo Clin 37. Eherer AJ, FordtranlJS. Fecal osmotic gap and pH in experimental diarrhea of various 38. Shiau YF, Feldman GM,lResnick MA, Coff PM. Stool electrolyte and osmolality Proc. 2012;87(6):596-602 causes. Gastroentero ogy. 1992;103(2):545-551. measurements in the eva uation of diarrheal disorders. Ann Intern Med. 1985;102(6):773-775.
-
Topazian M, Binder HJ. Brief report: factitious diarrhea detected by measurement of stool osmolality. N Engl J Med. 1994;330(20):1418-1419.
-
Craig RM, Atkinson AJ Jr. D-xylose testing: a review. Gastroenterology. 1988;95(1):223-231.
-
Lomer MC, Parkes GC, Sanderson JD. Review article: lactose intolerance in clinical practice–myths and realities. Aliment Pharmacol Ther. 2008;27(2):93-103.
-
Quigley EM, Abu-Shanab A.,Small intestinal bacterial overgrowth. Infect Dis Clin North Am. 2010;24(4):943-959 viii-ix.
-
Fasano A, Catassi C. Clinical practice. Celiac disease. N Engl J Med. 2012;367(25):2419- 2426.
-
Lindkvist B. Diagnosis and treatment of pancreatic exocrine insufficiency. World J Gastroenterol. 2013;19(42):7258-7266.
-
Nightingale J, Woodward JM, Small Bowel and Nutrition Committee of the British Society of Gastroenterology. Guidelines for management of patients with a short
bowel. Gut. 2006;55 suppl 4:iv1-iv12. 46. Walters JR, Pattni SS. Managing bile acid diarrhoea. Therap Adv Gastroenterol.
336
2010;3(6):349-357. 47. Khokhar N, Gill ML. Tropical sprue: revisited. J Pak Med Assoc. 2004;54(3):133-134. 48. Srirajaskanthan R, Shanmugabavan D, Ramage JK. Carcinoid syndrome. BMJ. 2010;341:c3941.
- Milan SA, Yeo CJ. Neuroendocrine tumors of the pancreas. Curr Opin Oncol. 2012;24(1):46-55. . Ramsay DB, Stephen S, Borum M, Voltaggio L, Doman DB. Mast c
50Gastrointestinal disease. Gastroenterol Hepatol. 2010;6(12):772-777.ells in 51. Chey WD, Kurlander J, Eswaran S. Irritable bowel syndrome: a clinical review. JAMA. 2015;313(9):949-958.
-
Ogbonnaya KI, Arem R. Diabetic diarrhea. Pathophysiology, diagnosis, and management. Arch Intern Med. 1990;150(2):262-267.
-
Daher R, Yazbeck T, Jaoude JB, Abboud B. Consequences of dysthyroidism on the digestive tract and viscera. World J Gastroenterol. 2009;15(23):2834-2838.
-
Tian XP, Zhang X.;Gastrointestinal complications of systemic sclerosis. World J 55. Petre S, Shah IA, Gilani N. Review article: Gastrointestinal amyloidosis—clinical Gastroenterol. 2013 19(41):7062-7068. features, diagnosis and therapy. Aliment Pharmacol Ther. 2008;27(11):1006-1016.
-
Fox DJ, Khattar RS. Carcinoid heart disease: presentation, diagnosis, and management. Heart. 2004;90(10):1224-1228.
-
Burgess TE, Mansoor AM. Giant a waves. BMJ Case Rep. 2017;2017.
337
CHAPTER 16
338
Gastrointestinal Bleeding
339 Case: An 84-year-old woman with hypotension anAn 84-year-old woman.with coronary artery disease, hypertension, admission. She subsequently developed bloody diarrhea, prompting son, who has noticed that her needs have been increasing over the past weakness. The patient’s son recently left town for an extended period inHeart rate is 108 beats per minute, and blood pressure is d vascular dementia is admitted to the hospital with abdominal pain and bloody diarrhea The pain is located over the left lower quadrant and is cramping in quality. It began suddenly 2 days before evaluation. The patient has a history of dementia and lives with her few months, particularly owing to forgetfulness and generalized of time on a business trip and left her alone at home for the first time over a year. She has not had fever, sweats, or chills. 92/57 mm Hg. The jugular venous waveform rises above the clavicle only in Trendelenburg position. Abdominal examination is notable for minimal pain to deep palpation of the left lower quadrant. There is no rebound, guarding, or rigidity. Rectal examination reveals the
presence ofibright red blood in the rectal vault. Cross-sectional onic imaging of the abdomen shows segmental thickening of the col wall involv ng the splenic flexure (arrows, Figure 16-1).
FIGURE 16-1 (From Pope TL, Harris JH. Harris & Harris’ The Radiology of Emergency Medicine. 5th ed. Philadelphia, PA: Lippincott Williams & Wilkins; 2012.)
What is the most likely cause of Gastrointestinal bleeding in this patient?
What is GI bleeding refers to the extravasation of blood into the lumen of the GI tract. It can be overt (acute) or occult (chronic), Gastrointestinal and is sometimes obscure. Overt GI bleeding is visible to the naked eye and presents with hematemesis, coffee-ground (GI) bleeding? emesis, melena, or hematochezia. Occult GI bleeding is not visible to the patient or clinician but can be established with a positive stool guaiac test with or without the presence of iron-deficiency anemia. Obscure GI bleeding refers to either overt or occult GI bleeding when a source remains unidentified after upper and lower endoscopy.1 What is Hematemesis refers to vomiting fresh blood, indicative of acute and active GI bleeding.1 hematemesis? What is coffee- Coffee-ground emesis refers to vomitus consisting of dark blood that has the appearance of coffee grounds (a result of ground emesis? the oxidation of iron from gastric acid), indicating that the GI bleeding has slowed or stopped.1 What is melena? Melena refers to black tarry stool (Figure 16-2).1
340
What is hematochezia? How common is Gastrointestinal bleeding? What is the initial management of patients presenting with overt Gastrointestinal bleeding? What are the 2 anatomic categories of Gastrointestinal bleeding?
What anatomic landmark distinguishes the upper and lower Gastrointestinal tracts? What are the visible characteristics of overt upper and lower Gastrointestinal bleeding? Under what circumstance might upper gastrointestinal bleeding present with hematochezia? Under what circumstance might lower Gastrointestinal bleeding present with melena? What investigative modalities are helpful for identifying the source of overt Gastrointestinal bleeding? How can CT angiography be useful for the detection of
FIGURE 16-2 Characteristic black tarry stool of melena. (From Sherman SC. Atlas of Clinical Emergency Medicine. Philadelphia, PA: Wolters Kluwer Health; 2016.)
Hematochezia refers to the passage of red or maroon blood from the rectum.1 In the industrialized world, GI bleeding is a common clinical problem, resulting in up to 150 hospital admissions per 100,000 persons annually.1 Overt GI bleeding can be massive and life-threatening with an overall mortality rate of up to 10%. Before undergoing diagnostic evaluation, hemodynamically unstable patients should be adequately resuscitated with isotonic crystalloid solution (eg, normal saline) and blood products (overaggressive resuscitation may exacerbate bleeding in some cases). Hemoglobin concentration should be followed serially, as a significant drop may not initially be apparent in acute hemorrhage. Blood is typically transfused to maintain hemoglobin concentration ≥7 g/dL (or higher in patients with comorbidities such as coronary artery disease). Additional therapeutic interventions such as endoscopic hemostasis or surgery may be necessary.1,2 Causes of GI bleeding can involve the upper or lower GI tract.
Upper GI bleeding arises from a site above the ligament of Treitz at the duodenojejunal flexure; lower GI bleeding arises from a site below the ligament of Treitz.1
Overt upper GI bleeding typically presents with hematemesis, coffee-ground emesis, or melena; overt lower GI bleeding typically presents with hematochezia.1
Upper GI bleeding can present with hematochezia if the rate is brisk.1
Lower GI bleeding can present with melena if the source is in the small intestine or right colon. The stool may also appear maroon-colored and mixed with blood.1
The initial diagnostic modality of choice for overt GI bleeding is upper endoscopy or colonoscopy, depending on whether the clinical presentation suggests an upper or lower source. If a source of bleeding is not identified after upper and lower endoscopy (ie, obscure overt GI bleeding), diagnostic modalities such as computed tomography (CT) angiography, catheter angiography, or radionuclide imaging should be considered. Active bleeding must be present for these studies to be useful. Other options for obscure overt GI bleeding in hemodynamically stable patients with low- volume bleeding include push enteroscopy, deep small bowel enteroscopy, and capsule endoscopy.1
CT angiography is a noninvasive modality used to identify a source of active GI bleeding in hemodynamically stable patients when endoscopy is nondiagnostic. It requires an arterial bleeding rate ≥0.5 mL/min to be reliable. Under these circumstances, it is associated with excellent sensitivity (86%) and specificity (95%). It has the additional advantage of being able to identify the etiology of bleeding in some cases (eg, tumor). The lack of therapeutic capability is a
341
Gastrointestinal How can catheter angiography be useful for the detection of Gastrointestinal bleeding? bleeding?
disadvantage. However, it can be used to direct and plan definitive treatment.1 Catheter angiography is used to identify a source of active GI bleeding when endoscopy is not feasible (eg, hemodynamic instability) or nondiagnostic (eg, bleeding obscures view), particularly when the source is thought to be in the lower GI tract. It requires an arterial bleeding rate ≥0.5-1.5 mL/min to be reliable. Sensitivity is variable, but specificity is close to 100%. Disadvantages include the invasive nature of the study and potential complications. However, it has the advantage of diagnostic and therapeutic capabilities, allowing for the infusion of vasoconstrictive drugs and the performance of embolization.1
How can radionuclide imaging be useful for the detection of Gastrointestinal bleeding? What investigative modalities are helpful for identifying the source of occult Gastrointestinal bleeding? How can capsule endoscopy be useful for the detection of Gastrointestinal bleeding? How can push enteroscopy and deep enteroscopy be useful for the detection of Gastrointestinal bleeding?
Radionuclide imaging uses red blood cells that are labeled with a radioactive tracer to detect sites of bleeding. It is the most sensitive diagnostic modality for active GI bleeding, requiring a minimum arterial bleeding rate of only 0.1 mL/min. The main limitation of radionuclide imaging is poor anatomic localization of the site of bleeding.1
Like overt GI bleeding, the initial diagnostic modalities of choice for occult GI bleeding are upper endoscopy and/or colonoscopy, which together identify the culprit lesion in up to one-half of cases. When initial endoscopic studies fail to identify a source, repeat examinations should be considered. The most common site of obscure occult GI bleeding is the small intestine, which should be evaluated further with capsule endoscopy, push enteroscopy, or deep small bowel enteroscopy (eg, double balloon enteroscopy). CT or magnetic resonance enterography may be considered in some cases, particularly when capsule endoscopy and deep enteroscopy are nondiagnostic.1
Capsule endoscopy is a simple, noninvasive modality that evaluates the small intestine in the setting of obscure GI bleeding. A capsule containing a tiny camera is swallowed by the patient and takes photographs of the GI tract as it passes through. It is generally the initial diagnostic modality of choice for the evaluation of obscure occult GI bleeding, identifying the culprit lesion in more than one-half of patients. The main disadvantage of capsule endoscopy is the lack of therapeutic capability.1 Push enteroscopy can reach as far as the proximal 60 to 80 cm of the jejunum. However, it has largely been replaced by deep enteroscopy, which reaches the distal small bowel. The diagnostic yield of deep enteroscopy is comparable with that of capsule endoscopy, but it is more invasive with higher risk. The therapeutic capabilities of push enteroscopy and deep enteroscopy provide an advantage over capsule endoscopy.1
342
Upper Gastrointestinal Bleeding
Which biochemical laboratory abnormality can provide a clue to the presence of upper Gastrointestinal bleeding? Which bedside procedure can confirm the presence of upper Gastrointestinal bleeding when it is suspected but not clinically apparent? What is the initial diagnostic modality of choice for upper Gastrointestinal bleeding?
What are the anatomic subcategories of upper Gastrointestinal bleeding?
Digested blood is a source of urea, which raises the blood urea nitrogen (BUN) level, leading to an Increased BUN- to-creatinine ratio in patients with upper GI bleeding.1
Red blood or coffee-ground aspirate from nasogastric lavage can confirm the presence of upper GI bleeding.
Most causes of upper GI bleeding can be definitively diagnosed with upper endoscopy. It should be performed within 24 hours in most patients with overt bleeding after adequate fluid resuscitation. Endoscopy may also be therapeutic in many cases. Conventional endoscopic techniques used to achieve hemostasis include injection therapy (eg, epinephrine, tissue adhesives), mechanical therapy (eg, clips, band ligation), and thermal therapy, including contact techniques (eg, electrocoagulation, thermocoagulation) and noncontact techniques (eg, argon plasma coagulation, laser photocoagulation). The method of choice depends on the nature of the target lesion.3 Upper GI bleeding can be esophageal, gastric, or duodenal.
343
Esophageal Causes of Upper Gastrointestinal Bleeding
What are the esophageal causes of upper Gastrointestinal bleeding?
A 48-year-old Variceal hemorrhage. man with a history of cirrhosis and portal hypertension presents with acute-onset massive hematemesis. A 54-year-old Mallory-Weiss tear. man with active alcohol abuse presents with hematemesis that began shortly after an episode of heavy retching. This condition Esophagitis. can be caused by a variety of processes, from swallowing pills to infection. Risk factors Esophageal cancer. include smoking tobacco, alcohol use, and gastroesophageal reflux disease.
What is the acute management of esophageal variceal hemorrhage?
What is the acute management of a Mallory-Weiss tear? What are the clinical features of Gastrointestinal bleeding caused by esophagitis? What are the clinical features of Gastrointestinal bleeding caused by esophageal
Gastroesophageal varices are portosystemic collaterals that develop as a result of portal hypertension and are present in up to one-half of patients with cirrhosis at the time of diagnosis. Acute esophageal variceal hemorrhage is a life-threatening condition associated with a high mortality rate. It accounts for the vast majority of GI bleeding in cirrhotics. In addition to basic resuscitative measures, patients should be treated with a vasoconstrictor (eg, octreotide) to reduce portal blood flow. In those with cirrhosis, short-term prophylactic antibiotics (eg, ceftriaxone) should be added. Pharmacologic therapy should coincide with endoscopic techniques such as balloon tamponade and band ligation. When bleeding cannot be controlled with the aforementioned strategies, transjugular intrahepatic portosystemic shunt (TIPS) may be helpful. A nonselective β-blocker (eg, propranolol) can help prevent future bleeding.4 Mallory-Weiss tears account for up to 15% of upper GI bleeding. Most episodes stop spontaneously with rebleeding rates of up to 10%. Hemostasis can be achieved endoscopically with bipolar electrocoagulation, epinephrine injection, clips, or band ligation.3 Esophagitis accounts for approximately 10% of upper GI bleeding; most patients have underlying gastroesophageal reflux disease. It typically presents with hematemesis. Patients tend to experience less hemodynamic instability compared with other causes of upper GI bleeding. Prognosis is excellent overall with a low risk of rebleeding.5
Esophageal cancer typically presents with progressive mechanical dysphagia and weight loss. Overt GI bleeding is uncommon. It tends to be occult, presenting with iron-deficiency anemia. However, overt bleeding can occur when a tumor erodes into nearby vascular structures, such as the aorta.6
344
cancer?
345
Gastric Causes of Upper Gastrointestinal Bleeding
What are the gastric causes of upper Gastrointestinal bleeding?
Responsible for most cases of upper GI bleeding. Gastric epithelial cell injury associated with inflammation. Gastric epithelial cell injury without inflammation. A manifestation of portal hypertension. A 75-year-old Japanese man with an extensive history of cigarette smoking presents with epigastric abdominal pain, weight loss, and melena. “Watermelon stomach.” A rare cause of upper GI bleeding, this lesion is characterized by a single large tortuous arteriole in the submucosa of the stomach. Associated with a hiatal hernia.
Peptic ulcer disease (PUD).3
Gastritis.
Gastropathy.
Gastric varices. Gastric cancer.
Gastric antral vascular ectasia (GAVE). Dieulafoy’s lesion.
Cameron lesion.
In addition to the Intravenous infusion of a proton pump inhibitor should be started immediately in patients suspected of GI usual resuscitation bleeding from PUD. Diagnostic endoscopy should be performed within 24 hours in most patients. Endoscopic measures, what is the hemostasis (eg, thermal therapy, epinephrine injection, clips) is indicated for ulcers with active bleeding (eg, acute management of spurting or oozing) or nonbleeding visible vessels; it is unnecessary for ulcers with a clean base or flat pigmented Gastrointestinal spot. After achieving hemostasis, intravenous proton pump inhibitor therapy should be continued for 72 hours bleeding caused by before transitioning to an oral agent.2 peptic ulcer disease? What common Helicobacter pylori affects nearly half of the world’s population and is associated with conditions such as gastritis, bacterial infection is a PUD, mucosa-associated lymphoid tissue (MALT) lymphoma, and gastric cancer. Noninvasive options for leading cause of diagnosing active infection include the urea breath test and stool antigen test. Active infection can also be gastritis? detected via endoscopic gastric biopsy with histology, culture, polymerase chain reaction, and rapid urease test. Various combinations of antibiotics (eg, clarithromycin and amoxicillin) and antisecretory agents (eg, proton pump inhibitors) are used to treat the infection. Diagnostic tests should be repeated after treatment to confirm eradication.7 What kind of Portal hypertensive gastropathy (PHG) is common in patients with cirrhotic and noncirrhotic portal hypertension. gastropathy is The diagnosis is made endoscopically, usually based on characteristic gross appearance of the gastric mucosa (a common in patients mosaic-like pattern resembling snake skin). PHG can cause both overt and occult GI bleeding. The treatment for with cirrhosis? overt bleeding is similar to that of variceal hemorrhage, including a vasoconstrictor (eg, octreotide) and prophylactic antibiotics in patients with cirrhosis. TIPS can be considered in some patients but is usually ineffective. Once the episode has resolved, a nonselective β-blocker (eg, propranolol) can help prevent future bleeding.8 How does the acute Gastric varices are present in one-fifth of patients with cirrhosis, often occurring along with esophageal varices. management of gastric Management of bleeding gastric varices is similar to that of esophageal varices, except obturation with tissue
346
variceal hemorrhage differ from esophageal hemorrhage? What are the risk factors for gastric adenocarcinoma? What are the clinical differences between gastric antral vascular ectasia and portal hypertensive gastropathy? In addition to the usual resuscitation measures, what is the acute management of Gastrointestinal bleeding caused by Dieulafoy’s lesions? What are Cameron lesions?
adhesives (eg, N-butyl-2-cyanoacrylate glue) is more effective than ligation for gastric varices. Procedures such as balloon-occluded retrograde transvenous obliteration (BRTO), TIPS, and surgery may be necessary.4
The strongest risk factor for gastric adenocarcinoma is Helicobacter pylori infection; others include cigarette smoking, alcohol use, male sex, a diet rich in salt and lacking in fruits and vegetables, and pernicious anemia.9 Like PHG, GAVE is diagnosed based on endoscopic appearance. GAVE is characterized by the presence of linear red stripes separated by normal mucosa (resembling a watermelon). It occurs in patients with portal hypertension but, unlike PHG, is also associated with autoimmune and connective tissue diseases, such as atrophic gastritis, systemic sclerosis, and pernicious anemia. Unlike PHG, pharmacologic management (eg, octreotide) is unhelpful in the setting of overt bleeding; endoscopy with argon plasma coagulation is the treatment of choice.8 Dieulafoy’s lesions are more common in men and account for up to 5% of upper GI bleeding. Often difficult to locate endoscopically, particularly when bleeding has stopped, Dieulafoy’s lesions can be a source of obscure GI bleeding. Overt bleeding is often massive and life-threatening. Endoscopic hemostasis using any of a variety of techniques (eg, epinephrine injection with electrocoagulation) is the treatment of choice.10
Cameron lesions describe linear gastric erosions and ulcers within the distal neck of a hiatal hernia, often associated with gastroesophageal reflux disease. The lesions are typically asymptomatic but can cause overt or occult bleeding, the latter of which is more common and may present with iron-deficiency anemia. Diagnosis and management are endoscopic.11
347
Duodenal Causes of Upper Gastrointestinal Bleeding
What are the duodenal causes of upper Gastrointestinal bleeding?
A 48-year-old man with months of epigastric abdominal pain that tends to improve after eating presents with melena and light-headedness. Inflammation of the duodenum from any cause. Types of vascular anomalies.
What are the strongest risk factors for peptic ulcer disease? What are some causes of duodenitis? What is angiodysplasia of the Gastrointestinal tract?
What hereditary disease is associated with telangiectasias of the Gastrointestinal tract?
Peptic ulcer disease.
Duodenitis. Angiodysplasia (ie, vascular ectasia, arteriovenous malformation, angiectasia) and telangiectasia. These entities also occur elsewhere in the GI tract.
Infection with Helicobacter pylori and use of nonsteroidal anti-inflammatory drugs (NSAIDs) are the strongest risk factors for gastric and duodenal ulcers. Duodenal ulcers cause overt GI bleeding more often than gastric ulcers because of proximity to the gastroduodenal artery. Duodenitis is often identified endoscopically or by imaging. Causes are similar to those of gastritis or gastropathy and include NSAID use, alcohol use, infection (eg, Helicobacter pylori), Crohn’s disease, celiac disease, and external beam radiation. Angiodysplasia refers to abnormal, ectatic, dilated, tortuous, and usually small (<10 mm) blood vessels that arise within the mucosal or submucosal layers of the GI tract. It is the most common vascular malformation of the GI tract, typically affecting patients >60 years of age. Lesions can be asymptomatic or result in overt or occult GI bleeding. Endoscopy is the diagnostic and therapeutic modality of choice.12 Hereditary hemorrhagic telangiectasia (HHT) is frequently associated with telangiectasias of the GI tract. Up to one-third of patients with HHT have experienced overt GI bleeding (hematemesis or melena), most often affecting patients in the fourth or fifth decades of life. Culprit lesions can be treated endoscopically (eg, laser therapy). Epistaxis is common in patients with HHT and can be confused for GI bleeding when blood is swallowed.13
348
Lower Gastrointestinal Bleeding
What is the initial diagnostic modality of choice for lower Gastrointestinal bleeding? What are the general subcategories of lower Gastrointestinal bleeding?
Most causes of lower GI bleeding can be definitively diagnosed with colonoscopy. It should generally be performed within 24 hours in most patients with overt lower GI bleeding after adequate fluid resuscitation. Colonoscopy may also be therapeutic in many cases. Conventional endoscopic techniques used to achieve hemostasis include injection therapy, mechanical therapy, and thermal therapy. The method of choice depends on the nature of the target lesion.14 Lower GI bleeding can be structural, vascular, or inflammatory.
349
Structural Causes of Lower Gastrointestinal Bleeding
What are the structural causes of lower Gastrointestinal bleeding?
A 28-year-old man with Crohn’s disease presents with painful defecation and bright red blood on the toilet paper. Older adults with painless hematochezia. These lesions may be benign, premalignant, or malignant. A 68-year-old woman presents with a change in bowel pattern, iron-deficiency anemia, weight loss, and hematochezia.
Anal fissure.
Diverticulosis. Colorectal polyps.
Malignancy.
What are the clinical An anal fissure is a tear in the mucosa of the anal canal. It typically presents with painful defecation and characteristics of anal red blood streaks on the surface of stool or toilet paper. Most fissures occur along the posterior midline; fissures? fissures that are not midline suggest an underlying condition such as anal cancer, Crohn’s disease, or syphilis. First-line therapies for acute and chronic fissures include Increased water and fiber intake, and sitz baths.15 What is the usual treatment Diverticulosis is the most common cause of lower GI bleeding requiring hospitalization. Like most causes for diverticular bleeding? of lower GI bleeding, it is typically self-limited. However, endoscopic hemostasis is recommended when active bleeding or stigmata of recent bleeding (eg, visible vessel, clot) are present.14 What should be suspected in Postpolypectomy bleeding accounts for just fewer than 10% of lower GI bleeding. However, incidence is on a patient who presents with the rise likely because of the Increased use of antiplatelet and anticoagulant medications. Endoscopy can be hematochezia after a recent used to achieve hemostasis.14 screening colonoscopy? What types of Colon cancer most often presents with occult GI bleeding, resulting in anemia and fatigue, but overt GI Gastrointestinal bleeding bleeding occasionally occurs. Lesions in the rectum frequently cause overt hematochezia. Colorectal cancer occur in patients with complicated by GI bleeding usually requires surgical management.16 colorectal cancer?
350
Vascular Causes of Lower Gastrointestinal Bleeding
What are the vascular causes of lower Gastrointestinal bleeding?
A 22-year-old man who is an avid weight lifter presents with intermittent hematochezia. These acquired lesions are a common source of lower GI bleeding in elderly patients. A 44-year-old woman with tight, shiny skin, dysphagia, Raynaud’s phenomenon, and intermittent hematochezia. Benign tumor of vascular origin. Consider this malignant vascular tumor in patients with acquired immunodeficiency syndrome (AIDS).
What is the management of bleeding internal hemorrhoids that are refractory to conservative measures (eg, Increased water and fiber intake)? What conditions are associated with bleeding from angiodysplasia? What are the characteristics of Gastrointestinal telangiectasias associated with systemic sclerosis? What are the characteristics Gastrointestinal hemangiomas?
What are the characteristics of Kaposi sarcoma of the Gastrointestinal tract?
Hemorrhoids.
Angiodysplasia.
Telangiectasias associated with systemic sclerosis (ie, scleroderma).
Hemangioma. Kaposi sarcoma.
Internal hemorrhoids are the second most common cause of lower GI bleeding requiring hospitalization. Rubber band ligation is commonly used to treat bleeding internal hemorrhoids. Complete evaluation of the colon with colonoscopy should be considered, particularly in older patients.14
Conditions associated with bleeding from angiodysplasia include aortic stenosis (Heyde’s syndrome), von Willebrand disease, and chronic kidney disease. The endoscopic technique of choice to establish hemostasis is argon plasma photocoagulation.12 Telangiectasias are the most common vascular anomaly associated with systemic sclerosis and can occur virtually anywhere in the GI tract. Bleeding events are often recurrent and severe, but occult GI bleeding also occurs frequently. Therapeutic options are limited; endoscopic and surgical treatments are associated with variable results.17 Hemangiomas are benign vascular tumors that infrequently involve the GI tract. Most patients experience painless and recurrent GI bleeding, usually presenting as large-volume hematochezia. A significant number of patients experience occult bleeding that manifests with iron-deficiency anemia. Diagnosis may be established with various imaging modalities and endoscopy. Treatment options include a variety of endoscopic and surgical techniques, which are often successful.18 Kaposi sarcoma is the most common GI malignancy in patients with AIDS. Forms of Kaposi sarcoma also occur in non-HIV populations, such as posttransplant patients on chronic immunosuppressive medication, and certain indigenous populations (eg, elderly Eastern European and Mediterranean men). It can affect virtually any part of the GI tract. Most lesions are asymptomatic, however both overt and occult bleeding can occur. The diagnosis is made endoscopically and confirmed with histology.19
351
Inflammatory Causes of Lower Gastrointestinal Bleeding
How does bleeding manifest in patients with inflammatory causes of lower Gastrointestinal bleeding?
Inflammatory causes of lower GI bleeding often present with bloody diarrhea.1
What are the inflammatory causes of lower Gastrointestinal bleeding?
Often precipitated by Ischemic colitis. hypotension. Dozens of patrons of a Infectious colitis. particular fast-food restaurant develop bloody diarrhea. A 26-year-old man Inflammatory bowel disease. presents with chronic abdominal pain, weight loss, and bloody diarrhea and is found to have erythema nodosum (see Figure 15-3). Related to the Radiation coloproctitis. treatment for a variety of malignancies, including prostate, cervical, and rectal cancer.
What findings on plain abdominal radiography can be suggestive of ischemic colitis?
What should be suspected in a patient who develops bloody
Abdominal CT imaging is the modality of choice in patients suspected of ischemic colitis. However, a plain abdominal radiograph may be obtained in the general evaluation of abdominal pain. Severe cases of ischemia are associated with findings such as pneumatosis intestinalis and “thumbprinting,” which is the result of submucosal edema (Figure 16-3).20
FIGURE 16-3 An abdominal radiograph showing the “thumbprinting” sign (arrows) of ischemic colitis. (From Riddell R, Jain D. Lewin, Weinstein and Riddell’s Gastrointestinal Pathology)and its Clinical Implications. 2nd ed. Philadelphia, PA: Wolters Kluwer Health; 2017.
Hemolytic uremic syndrome can be a complication of infectious colitis caused by invasive organisms, particularly Escherichia coli O157:H7.
352
diarrhea followed by hemolytic anemia, thrombocytopenia, and Acute Kidney Injury? Which type of inflammatory bowel disease is more commonly associated with hematochezia?
What are the clinical features of radiation coloproctitis?
The vast majority of patients with ulcerative colitis experience hematochezia. The clinical features of Crohn’s disease are more variable; most often characterized by chronic nonbloody diarrhea with abdominal pain and weight loss. Lower endoscopy with biopsy is diagnostic of both subtypes. Ulcerative colitis involves the rectum and colon, and is characterized by continuous mucosal and submucosal inflammation. Crohn’s disease can affect any part of the GI tract and is characterized by noncontinuous transmural inflammation (ie, “skip” lesions). A small proportion of patients have features of both, referred to as “unclassified” inflammatory bowel disease.21 The risk of developing coloproctitis after regional external beam radiation therapy correlates with the field size and dose of radiation. Acute coloproctitis presents within 90 days of therapy with symptoms such as diarrhea with or without blood, nausea, cramps, tenesmus, urgency, and mucus discharge. Chronic coloproctitis can present during the acute period but more often presents months to years later. Symptoms are similar to acute coloproctitis but may also include severe bleeding, strictures, fistula formation, bowel obstruction, and perforation. Bleeding complications are addressed endoscopically or surgically.22
353 Case Summary with left lower quadrant abdominaltpain, diarrhea, and hematochezia, have colonic inflammation involving the splenic flexure on cross- An 84-year-old woman with a his ory of vascular disease presents in the context of poor oral intake and hypotension, and is found to sectional imaging.
What is the most likely cause of Gastrointestinal bleeding in this patient? Ischemic colitis.
354
are present in this case? Bonus Questions
What risk factors for ischemic colitis
ischemic colitis more likely than other What features of this case make causes of inflammatory lower Gastrointestinal bleeding? fulminantigangrenous ischemic colitis? What clin cal features are suggestive of
What isithe role of endoscopy in the How should the patient in this case be diagnos s of ischemic colitis? treated?
Risk factors for ischemicfcolitis include increasing age and comorbidities, particularly vascular disease. The patient in this case most likely hasn 20 atherosclerotic disease o the mesenteric arteries, which makes the colon susceptible to ischemia, particularly when there is an abrupt change i perfusion pressure. Abrupt onset of symptoms is characteristic of ischemic colitis; symptoms of infectious colitis or inflammatory bowel disease tend to be more
insidious in onset. On cross-sectional imaging, segmental involvement of the GI tract in a vascular or “watershed” distribution—in this case the splenic flexure (see arrows)—is suggestive of ischemia. Figure 16-1, 20 Fulminant gangrenous ischemic colitis occurs in a subset of patients with ischemic colitis when there is severe and prolonged ischemia that results in transmural necrosis, leading to complications such as bowel perforation. Physical findings of peritonitis, including rebound tenderness, involuntary guarding, and abdominal rigidity, are suggestive. The identification of peritoneal free air on abdominal imaging provides additional evidence. 20 Lower endoscopy can confirm the diagnosis of ischemic colitis via direct visualization and biopsy and provides an assessmentiof severity, which has implications on prognosis and treatment. The presence of cyanosis and pseudopolyps are suggestive of transmural schemia. 20 Aggressive fluid resuscitation to restore favorable hemodynamics is the cornerstone of managing uncomplicated ischemic colitis. Most patients improve with these measures alone. Complicated cases may require surgical intervention. 20
355 Key Points
GI bleeding can be overt (acute) or occult (chronic), and is sometimes obscure.
Overt GI bleeding is visibly evident to the patient or clinician and hematochezia. presents with hematemesis, coffee-ground emesis, melena, or Occult GI bleeding is not visibly evident but can be established with a positive stool guaiac test, with or without the presence of Obscure GI bleeding refers to recurrent GI bleeding (either overt or iron-deficiency anemia. occult) in which a source remains unidentified after upper and lower endoscopy.
Patients with overt GI bleeding andlhemodynamic instability must Upper GI bleedingiis generally associated with hematemesis, . Lower GI bleeding is generally associated with hematochezia. therapeutic) modality of choice in the setting of GI bleeding. Lower GI bleeding can be structural, vascular, or inflammatory. be resuscitated with isotonic crystal oid solutions and blood products. Causes of GI bleed ng can involve the upper or lower GI tract coffee-ground emesis, and/or melena. Upper or lower endoscopy is the initial diagnostic (and often Upper GI bleeding can be esophageal, gastric, or duodenal.
356
References 1. Kim BS, Li BT, Engel A, et al. Diagnosis of Gastrointestinal bleeding: a practical guide for clinicians. World J Gastrointest Pathophysiol. 2014;5(4):467-478.
-
Laine L, Jensen DM. Management of patients with ulcer bleeding. Am J Gastroenterol. 2012;107(3):345-360; quiz 61.
-
Szura M, Pasternak A. Upper non-variceal Gastrointestinal bleeding—review the effectiveness of endoscopic hemostasis methods. World J Gastrointest Endosc.
2015;7(13):1088-1095. 4. Garcia-Tsao G, Bosch J. Management of varices and variceal hemorrhage in cirrhosis. N Engl J Med. 2010;362(9):823-832. Guntipall P, Chason R, Elliott A, Rockey DC. Upper Gastrointestinal bleeding caused
-
by severeiesophagitis: a unique clinical syndrome. Dig Dis Sci. 2014;59(12):2997-3003. 6. Rustgi AK, El-Serag HB. Esophageal carcinoma. N Engl J Med. 2014;371(26):2499-2509. 7. of Helicobacter pylori diagnosis, treatment, and methods to detect eradication..World Jw Garza-Gonzalez E, Perez-Perez GI, Maldonado-Garza HJ, Bosques-Padilla FJ A revie Gastroenterol. 2014;20(6):1438-1449.
-
Cubillas R, Rockey DC. Portal hypertensive gastropathy: a review. Liver Int. 2010;30(8):1094-1102.
-
ThrumurthytSG, Chaudry MA, Hochhauser D, Mughal M. The diagnosis and 10. Chaer RA, Helton WS. Dieulafoy’s disease. J Am Coll Surg. 2003;196(2):290-296. managemen of gastric cancer. BMJ. 2013;347:f6367.
-
Kapadia S, Jagroop S, Kumar A. Cameron ulcers: an atypical source for a massive upper Gastrointestinal bleed. World J Gastroenterol. 2012;18(35):4959-4961.
-
Sami SS, Al-Araji SA, Ragunath K. Review article: Gastrointestinal angiodysplasia— pathogenesis, diagnosis and management. Aliment Pharmacol Ther. 2014;39(1):15-34.
-
Kjeldsen AD, Kjeldsen J. Gastrointestinal bleeding in patients with hereditary hemorrhagic telangiectasia. Am J Gastroenterol. 2000;95(2):415-418.
-
Ghassemi KA, Jensen DM. Lower GI bleeding: epidemiology and management. Curr Gastroenterol Rep. 2013;15(7):333.
-
Steele SR, Madoff RD. Systematic review: the treatment of anal fissure. Aliment Pharmacol Ther. 2006;24(2):247-257.
-
De Rosa M, Pace U, Rega D, et al. Genetics, diagnosis and management of colorectal cancer (review). Oncol Rep. 2015;34(3):1087-1096.
-
Duchini A, Sessoms SL. Gastrointestinal hemorrhage in patients with systemic sclerosis and CREST syndrome. Am J Gastroenterol. 1998;93(9):1453-1456.
-
Tan MC, Mutch MG. Hemangiomas of the pelvis. Clin Colon Rectal Surg. 2006;19(2):94-101.
-
Arora M, Goldberg EM. Kaposi sarcoma involving the Gastrointestinal tract. Gastroenterol Hepatol. 2010;6(7):459-462.
-
Trotter JM, Hunt L, Peter MB. Ischaemic colitis. BMJ. 2016;355:i6600.
-
Mozdiak E, O’Malley J, Arasaradnam R. Inflammatory bowel disease. BMJ. 2015;351:h4416.
-
Do NL, Nagle D, Poylin VY.;Radiation proctitis: current strategies in management. Gastroenterol Res Pract. 2011 2011:917941.
357
358
CHAPTER 17
359
Hepatocellular Liver Injury
360 Case: A 37-year-old man with a skin rash A 37-year-old man with a history of allogeneic stem cell transplantation at 35 years of age for chronic myelogenous leukem a,
complicated by cutaneous graft-versus-host disease, is admitted toithe describes a new skin rash affecting his face, torso, arms, and legs thats medical care. Medications include prednisone 40 mg daily for graft-prophylaxis against Pneumocystis jirovecii pneumonia. There have been hospital with fever, skin rash, and abdominal pain. The patient started 1 week before admission. He has had fever and chills over thi time. The development of abdominal pain prompted him to seek versus-host disease and trimethoprim/sulfamethoxazole for no recent medication changes, and the patient does not take over-the- counter medications or herbal supplements. Temperature is 38.6°C. There is scleral icterus. Papu ar lesions in
various stages of healing are observed over the skin oflthe face, torso, unroofed, revealing serosanguinous fluid; others appear dry and and arms. Some of the lesions are flaccid vesicles that are easily crusted (Figure 17-1A and B). Tender hepatomegaly is present. Serum alanine aminotransferase (ALT) is 5340 U/L; aspartate aminotransferase (AST), 4325 U/L; alkaline phosphatase (ALP),
252 U/L; total bilirubin, 3.3 mg/dL; and international normalized ratio molecular identification of herpes simplex virus, which returns (INR), 2.8. A skin lesion (arrow, Figure 17-1A) is swabbed for negative.
FIGURE 17-1 (Courtesy of Jesse J. Keller, MD.)
What is the most likely cause of hepatocellular liver injury in this patient?
What biochemical laboratory pattern describes hepatocellular liver injury? What are the aminotransferases?
What are the sources of aminotransferases in the body? What is the definition of an abnormal
Diseases of the liver produce characteristic biochemical patterns of injury. Distinguishing hepatocellular liver injury from cholestatic liver injury can be helpful in narrowing the differential diagnosis. Hepatocellular liver injury refers to the predominance of serum aminotransferase elevation compared with serum ALP. Serum bilirubin levels may or may not be elevated.1 The aminotransferases, which include AST and ALT, are enzymes that catabolize amino acids, generating products that enter the Krebs cycle. AST and ALT are highly concentrated in the liver and are immediately leaked into the circulation when there is hepatocyte necrosis.1 AST and ALT are present in high concentrations in the liver. AST is also present in the tissues of the heart, skeletal muscle, kidney, and brain, and in red blood cells. There are low levels of ALT in skeletal muscle and kidney. Elevated serum ALT is generally more specific for liver damage than elevated AST.1 An abnormal aminotransferase level is defined by a value that exceeds the upper limit of normal (normal values vary by laboratory). Aminotransferase levels below the lower limit of normal are of no clinical significance.1
361
aminotransferase How is the normal value range determined for most level? laboratory tests? Using the 2 standard deviation rule, what percentage of healthy patients will have abnormal aminotransferase levels? How sensitive is aminotransferase elevation for the presence liver damage? How is the severity of aminotransferase elevation delineated?
What life-threatening condition can occur in patients with hepatocellular liver injury?
The causes of hepatocellular liver injury can be separated into which general categories?
Generally, the normal range for a given laboratory test is defined as the values within 2 standard deviations of the mean of the general population.
In a normal distribution, 95% of values lie within 2 standard deviations of the mean. This means that 2.5% of healthy individuals will have aminotransferase levels below normal, and 2.5% of healthy individuals will have aminotransferase levels above normal.1
Aminotransferase elevation is generally a sensitive marker of liver damage but may not be present in all patients. For example, some patients with chronic hepatitis C infection and others with nonalcoholic fatty liver disease have normal aminotransferase levels despite histologic evidence of liver damage.1
Severity of hepatocellular liver injury is variably defined, but the following thresholds provide a rule of thumb: Serum aminotransferase elevation is considered mild when it is <5 times the upper limit of normal, moderate when 5 to 10 times the upper limit of normal, and marked when >10 times the upper limit of normal. Marked aminotransferase elevation is usually the result of acute liver injury.1 Acute liver failure is a life-threatening condition that usually occurs in patients without preexisting liver disease and is characterized by biochemical laboratory evidence of acute liver injury and coagulopathy, jaundice, and encephalopathy over a period of days to months. Multiple organ failure and death occur in up to half of cases. In patients with acute liver injury, the risk of acute liver failure is generally low but can be significant in certain circumstances. For example, acute liver failure occurs in approximately 15% of pregnant women with hepatitis E infection.2,3 The causes of hepatocellular liver injury can be separated into the following categories: infectious, toxic, vascular, hereditary, and other.
362
Infectious Causes of Hepatocellular Liver Injury
What are the infectious causes of hepatocellular liver injury?
A vaccine is available for this virus that is transmitted via the fecal-oral route. A vaccine is not available for this virus that is transmitted via the fecal-oral route. In adult patients, this virus is most commonly acquired through sexual transmission. Often acquired when needles are shared between intravenous drug users. Infection with this virus occurs exclusively in patients with hepatitis B infection. This virus is responsible for causing infectious mononucleosis. In the immunocompetent host, infection with this virus is typically asymptomatic but can present with a syndrome similar to mononucleosis. Hepatitis caused by these viruses may be associated with characteristic cutaneous lesions.
What are the clinical features of hepatitis A virus infection?
Hepatitis A virus (HAV).
Hepatitis E virus (HEV).
Hepatitis B virus (HBV).
Hepatitis C virus (HCV).
Hepatitis D virus (HDV).
Epstein-Barr virus (EBV).
Cytomegalovirus (CMV).
Herpes simplex virus (HSV) and varicella-zoster virus (VZV).
HAV infection is found worldwide, endemic in areas with poor sanitation. Most infected adults are symptomatic. The preicteric phase is characterized by symptoms that develop 15 to 50 days (mean, 30 days) after exposure and include anorexia, nausea and vomiting, malaise, fever, headache, and abdominal pain. This is followed by the onset of the icteric phase, characterized by dark urine, acholic stools, and jaundice. Aminotransferases are usually markedly elevated, often >1000 U/L. Extrahepatic manifestations can occur (eg, arthralgias). Diagnosis is established by detecting serum immunoglobulin M (IgM) anti-HAV antibodies. The illness is usually self-limited, and the vast majority of patients fully recover within weeks to months.4
What are the clinical features of hepatitis E virus infection?
HEV infection is found worldwide, endemic in areas with poor sanitation. The incubation period ranges from 20 to 60 days (mean, 40 days). Most patients are asymptomatic. Those with symptoms experience a syndrome similar to that of HAV infection, including the preicteric phase, icteric phase, and potential extrahepatic manifestations. Aminotransferases are usually markedly elevated, often >1000 U/L. There are a variety of diagnostic serum assays, each with variable performance characteristics; a combination of tests may ultimately be needed to confirm the diagnosis. In healthy populations, the illness is usually self-limited, and the vast majority of patients fully recover within weeks to
363
months. However, there is a risk of acute liver failure, particularly in pregnant women.5 What is the HBV infection is acquired via contact with infected blood or body fluid. In the industrialized world, adults most natural history commonly acquire the infection via sexual contact. Most adults are symptomatic, and clinical manifestations are similar of acute to other forms of acute viral hepatitis. Aminotransferases are usually markedly elevated, often >1000 U/L. In healthy hepatitis B virus patients, the illness is self-limited in the vast majority of cases. It progresses to acute liver failure in <1% of adults, which infection? is associated with a mortality rate of approximately 80% without liver transplantation. Chronic HBV infection develops in <5% of adult patients.6 What is the HCV infection is most commonly acquired through blood transfusion or intravenous drug use. Most adults are natural history asymptomatic. Patients who are symptomatic experience a syndrome similar to other causes of acute viral hepatitis. of acute Aminotransferases are usually markedly elevated, often >1000 U/L. Acute HCV infection rarely causes acute liver hepatitis C virus failure. Although the illness can be self-limited, most patients develop chronic infection.7 infection? What is the Like HBV, HDV is transmitted via contact with infected blood or body fluid. Infection requires the presence of either natural history acute or chronic HBV infection. Acute coinfection typically results in more severe clinical manifestations compared with of acute acute HBV infection alone, and it is cleared along with HBV in the vast majority of cases. In contrast, superinfection in hepatitis D virus patients with chronic HBV infection usually results in chronic HDV infection, which is associated with more rapid infection? progression to cirrhosis.8 What are the Most healthy patients with EBV infection are asymptomatic, but some experience infectious mononucleosis, which is clinical features characterized by the constellation of fever, sore throat, and lymphadenopathy. Mild hepatocellular liver injury is found of Epstein-Barr in most patients with mononucleosis. Serologic tests (eg, heterophile antibody test) are used to establish the diagnosis. virus hepatitis? Some patients (particularly those who are immunocompromised) develop severe hepatitis. Histologic findings include vacuolization of the hepatocytes and periportal infiltration with lymphocytes and monocytes. Antiviral therapy (eg, ganciclovir) may be effective in severe disease.9,10 What are the Most healthy patients with CMV infection are asymptomatic, but some experience a self-limited mononucleosis-like clinical features illness. Immunocompromised patients are at risk for primary infection or reactivation of dormant infection, which can of result in tissue-invasive disease, including hepatitis. The spectrum of severity can range from mild to acute liver failure. cytomegalovirus Definitive diagnosis of CMV hepatitis requires histopathologic identification of characteristic inclusion bodies (ie, Owl’s hepatitis? eye appearance) or viral elements by immunohistochemical or molecular techniques. Ganciclovir is the antiviral agent of choice for severe disease.10-12 What are the HSV hepatitis is a life-threatening condition associated with disseminated HSV-1 and HSV-2 infections. It accounts for clinical features <10% of viral causes of acute liver failure. Immunocompromised patients and women in the third trimester of of herpes pregnancy are at highest risk. Aminotransferases are often 100 to 1000 times the upper limit of normal. In some cases, simplex virus disseminated HSV involves the skin, producing vesicular or pustular lesions in a generalized distribution, which evolve hepatitis? into dried crusted papules and eventually shallow punched-out ulcers. On clinical examination, lesions are often found in various stages of healing. Diagnosis of HSV hepatitis is based on histopathologic identification of characteristic intranuclear inclusion bodies or viral elements by immunohistochemical or molecular techniques. Intravenous acyclovir is the antiviral agent of choice.10,12 What are the Immunocompromised patients are at risk for disseminated VZV infection, which typically occurs as a result of clinical features reactivation of dormant infection. Hepatic involvement usually leads to acute liver failure. Cutaneous lesions are of varicella- frequently present, which are clinically indistinguishable from disseminated cutaneous HSV. The diagnosis is zoster virus confirmed through the identification of VZV from skin lesions or affected organs. Intravenous acyclovir is the antiviral hepatitis? agent of choice.10 Acute viral hepatitis is generally associated with ALT:AST ratio >1.13
364
Toxic Causes of Hepatocellular Liver Injury
What are the toxic causes of hepatocellular liver injury?
AST:ALT ratio >2:1. A 33-year-old woman with depression and prior suicide attempts presents with nausea, vomiting, and marked hepatocellular liver injury. A 22-year-old woman is admitted to the intensive care unit with acute liver failure after a weekend of partying. Patients with a history of repeated blood transfusions are at risk. Makes Mario bigger and faster.
Alcohol. Acetaminophen overdose.
Recreational drug use.
Iron overload (ie, secondary hemochromatosis).
Mushrooms.
What are the Alcoholic hepatitis can develop as acute or acute-on-chronic liver injury. Fever, jaundice, and tender characteristic hepatomegaly are the hallmarks of the condition. Unlike most causes of toxic hepatitis, aminotransferase levels biochemical laboratory rarely exceed 300 U/L in alcoholic hepatitis. The AST:ALT ratio is typically >2. Alkaline phosphatase is features of alcoholic frequently elevated, and total bilirubin is elevated at least 5-10 times the upper limit of normal, although it can hepatitis? be >10 times the upper limit of normal in some cases. Because of the limited degree of aminotransferase elevation, patients may have a cholestatic pattern of liver injury.1,14 What medications are Virtually any medication can lead to liver injury. Acetaminophen is one of the most common causes of acute most commonly liver failure, particularly in young previously healthy patients. Other medications that cause acute liver failure implicated in acute with relatively high frequency include isoniazid, propylthiouracil, phenytoin, valproate, nitrofurantoin, and liver failure? ketoconazole. In patients with hepatocellular liver injury, it is important to carefully review prescription and over-the-counter medications, as well as herbal supplements.15 Which recreational The synthetic amphetamine 3,4-methylenedioxymethamphetamine (MDMA, “ecstasy,” or “molly”) is used as a drug frequently causes recreational drug for its euphoric effects. It can cause acute hepatitis and is responsible for a significant acute liver failure in proportion of cases of acute liver failure in young patients. There is marked aminotransferase elevation (often the industrialized >1000 U/L) along with marked hyperbilirubinemia. Management is supportive; transplantation may be world? necessary in some cases.16 What conditions are Secondary hemochromatosis can develop as a result of Increased intestinal absorption of iron or recurrent blood associated with transfusions. Associated conditions include hemoglobinopathies (eg, sickle cell disease, thalassemia), inherited secondary hemolytic anemia (eg, glucose-6-phosphate dehydrogenase deficiency, hereditary spherocytosis), and hemochromatosis? myelodysplasia. Which species of Known as the “death cap,” Amanita phalloides is responsible for most cases of acute liver failure caused by mushroom is most mushroom poisoning. Accidental ingestion by amateur mushroom hunters is the most frequent manner of frequently responsible intoxication. Clinical manifestations range from a mild subclinical presentation to acute liver failure. for acute liver failure Gastrointestinal symptoms (eg, abdominal pain, vomiting, diarrhea) typically appear 6 to 24 hours after and death? ingestion. This phase is usually followed by a period of apparent clinical improvement before the onset of marked aminotransferase elevation and acute liver failure.17 With the exception of alcoholic hepatitis, toxic causes of acute hepatitis are generally associated with ALT:AST ratio >1.13
365
Vascular Causes of Hepatocellular Liver Injury
What are the vascular causes of hepatocellular liver injury?
A 23-year-old woman is admitted to the hospital with hemorrhagic shock after a motor vehicle accident and is found to have aminotransferase elevation >75 times the upper limit of normal. “Nutmeg liver.” A 24-year-old woman with factor V Leiden thrombophilia who recently started oral contraceptives presents with acute abdominal pain and hepatocellular liver injury.
Ischemic hepatitis (ie, shock liver).
Congestive hepatopathy. Budd-Chiari syndrome.
What are the characteristic Ischemic hepatitis occurs as a result of inadequate perfusion of the liver. Aminotransferase elevation is biochemical laboratory marked, often >50 times the upper limit of normal. When hemodynamics are restored, there is often a features of ischemic hepatitis? rapid decrease in aminotransferase levels after the initial peak.1,14 What are the clinical features Congestive hepatopathy occurs as a result of elevated central venous pressure, most commonly related to of congestive hepatopathy? cardiomyopathy, pulmonary hypertension, constrictive pericarditis, or valvulopathy (eg, mitral stenosis, tricuspid regurgitation). Manifestations include hepatomegaly, jaundice, peripheral edema, pleural effusions, ascites, and splenomegaly. Mild hyperbilirubinemia (usually <3 mg/dL) is the most common laboratory abnormality; when present, aminotransferase elevation is only mild. Marked aminotransferase elevation in patients with decompensated heart failure is suggestive of ischemic hepatitis.18 What are the biochemical Acute Budd-Chiari syndrome is characterized by abdominal pain, hepatomegaly, ascites, and laboratory features of Budd- hepatocellular liver injury that is at least moderate in severity. Acute liver failure is uncommon. In Chiari syndrome? subacute and chronic forms of the disease, aminotransferase levels may be normal or only mildly elevated.19 Vascular causes of acute hepatitis are generally associated with AST:ALT ratio >1.13
366
Hereditary Causes of Hepatocellular Liver Injury
What are the hereditary causes of hepatocellular liver injury?
Men with this condition are usually diagnosed at a younger age than women. A 26-year-old man presents with hepatocellular liver injury, hemolytic anemia, delusions, and hallucinations. Associated with emphysema.
How sensitive is aminotransferase elevation in patients with hereditary hemochromatosis? What is the spectrum of liver disease in patients with Wilson’s disease?
Hereditary hemochromatosis. Blood loss from menstruation is protective against iron overload.
Wilson’s disease.
α-1 Antitrypsin deficiency.
Hereditary hemochromatosis is an autosomal recessive condition and is common in whites. Aminotransferase levels are typically normal or only mildly Increased, even in patients who develop fibrotic liver disease. Elevated aminotransferase levels in these patients should prompt consideration of other causes of hepatocellular liver injury (eg, chronic viral hepatitis).20
Wilson’s disease is most commonly diagnosed in late childhood or adolescence, but it can present later in life. It should be suspected in patients with hepatocellular liver injury and concomitant hemolytic anemia, with or without psychiatric or neurologic symptoms. There is a wide spectrum of liver involvement, including asymptomatic aminotransferase elevation, acute hepatitis, acute liver failure, and progressive chronic liver disease. Low serum ceruloplasmin levels, elevated 24-hour urine copper excretion, and the presence of Kayser-Fleischer rings (a brown ring around the iris from copper deposition) (Figure 17-2) are consistent with the diagnosis. Liver biopsy is sometimes necessary to confirm the diagnosis.1,21
FIGURE 17-2 Brown copper deposition in the periphery of the cornea (Kayser-Fleischer ring) in a patient with Wilson’s disease. Rings are not often this pronounced. Milder cases may not be visible with the naked eye but can be identified with a slit lamp examination. (From Tasman W, Jaeger E. The Wills Eye Hospital Atlas of Clinical Ophthalmology. 2nd ed. Philadelphia, PA: Lippincott Williams & Wilkins; 2001.)
What is the α-1 Antitrypsin deficiency is most often diagnosed in childhood, but in adults it may present with pulmonary or mechanism of hepatic manifestations. The mechanisms of lung and liver disease differ. Increased proteolytic activity in the lung liver injury in the leads to the development of emphysema. In contrast, liver disease is the result of intrahepatic accumulation of setting of α-1 abnormally folded α-1 antitrypsin protein. Aminotransferase elevation is mild in most patients. The diagnosis can be antitrypsin confirmed with low serum α-1 antitrypsin levels and phenotype or genotype determination.22 defHereditary causes of acute hepatitis are generally associated with ALT:AST iciency? ratio >1.13
367
368
Other Causes of Hepatocellular Liver Injury
What are the other causes of hepatocellular liver injury?
Associated with the metabolic Nonalcoholic fatty liver disease (NAFLD). syndrome. A middle-aged woman with a Autoimmune hepatitis. history of Graves’ disease presents with marked hepatocellular liver injury and is found to have an elevated serum protein gap, positive antinuclear antibody, and positive anti–smooth muscle antibody. Cholestatic liver injury Acute biliary obstruction. eventually predominates in patients with this condition. Pregnancy. HELLP syndrome. Hepatocellular liver injury and Celiac disease. elevated tissue transglutaminase antibodies.
What are the clinical features NAFLD is the most common cause of hepatocellular liver injury in the industrialized world. Most of nonalcoholic fatty liver patients are asymptomatic or have nonspecific symptoms and the diagnosis is suspected because of mild disease? aminotransferase elevation. There is no specific blood test for NAFLD, and it is a diagnosis of exclusion. Liver biopsy can distinguish between simple steatosis (nonalcoholic fatty liver) and steatohepatitis (nonalcoholic steatohepatitis) with or without fibrosis.1 What are the clinical features Autoimmune hepatitis occurs globally in patients of various ages and ethnicities but is most common in of autoimmune hepatitis? women. It generally causes chronic and progressive hepatitis but may follow a fluctuating course with periods of Increased or Decreased disease activity. Presentation can vary from subclinical with mild hepatocellular liver injury, to acute liver failure with aminotransferases >1000 U/L. Some cases are characterized by a cholestatic pattern of liver injury. An elevated serum protein gap can be a clue to the diagnosis. The presence of serum antinuclear and anti–smooth muscle antibodies is characteristic. Immunosuppressive medication (eg, prednisone with or without azathioprine) is the cornerstone of treatment.23 What are the biochemical Although biliary obstruction will eventually cause a cholestatic pattern of liver injury, a hepatocellular laboratory features of acute pattern occurs early on. Aminotransferases are usually moderately elevated with an associated biliary obstruction? hyperbilirubinemia. The early hepatocellular predominance is explained by the fact that the aminotransferases are immediately leaked into the circulation upon hepatocyte necrosis. In contrast, ALP is synthesized in response to cholestasis and accumulating bile salts, which requires more time.1 What is HELLP syndrome? HELLP is an acronym that stands for Hemolysis, Elevated Liver enzymes, and Low Platelets. It is a syndrome that develops in pregnant women, usually between the 27th and 37th gestational weeks. The pathogenesis of HELLP syndrome is unclear but it may be a severe variant of preeclampsia. Aminotransferase elevation >2 times the upper limit of normal is required for the diagnosis. The presence of either AST >150 U/L or ALT >100 U/L is associated with a higher risk of serious maternal morbidity.24 How common is celiac disease Hepatocellular liver injury may be the only manifestation of celiac disease. Patients with unexplained in patients with unexplained hepatocellular liver injury should be tested for celiac disease, which is present in up to 10% of cases.1 he Theser“other” causes of hepatitis are generally associated with ALT:AST patocellula liver injury? ratio >1.13
369 Case Summary abdominal pain and is found to have diffuse cutaneous vesicular nd A 37-year-old immunocompromised man presents with fever a lesions and marked hepatocellular liver injury.
What is the most likely cause of hepatocellular liver injury in this patient? Disseminated varicella-zoster virus infection.
370
Bonus Questions
aminotransferase elevation aminotransferase elevation (>10 times the upper limit offnormal) is usually the result of acute liver injury, whichtmayibe caused by viral infection, drug Is the severity of In patients with hepatocellular liver injury, the degree o aminotransferase elevation can be helpful in narrowing he d fferential diagnosis. Marked helpful in establishing the toxicity (especially acetaminophen), ischemic hepatitis, autoimmune hepatitis, mushroom toxicity, or Wilson’s disease. 1 cause of hepatocellular liver injury? What historical information Disseminated VZV is significantly more common in immunocompromised patients. It typically manifests with cutaneous lesions with or without
makes disseminated varicella- visceral involvement (eg, hepatitis, pneumonitis, encephalitis). Visceral involvement may precede the development of cutaneous lesions in some cases.k zoster virus an important Disseminated VZV is the most frequent late infection of bone marrow transplantation recipients, and concurrent graft-versus-host disease is a major ris consideration in this case? factor. What other infectious cause of Disseminated VZV and HSV infections can both present with cutaneous vesicular lesions n a generalized distribution that are clinically 25 hepatocellular liver injury can indistinguishable from one another. To discriminatelbetween HSV and VZViinfection, theilesions can be swabbed anditested using molecular (eg, aking Should graft-versus-host Graft-versus-host disease of the liver should be considered in this case, particularly given the presence of cutaneous graft-versus-host disease. However, present with similar polymerase chain reaction) or immunohistochemica (eg, immunofluorescence) techniques. In this case, molecular test ng for HSV was negative, m cutaneous lesions? VZV the most likely diagnosis (testing for VZV should be performed to conf rm the diagnosis). 10 disease of the liver be the pattern of liver injury is typically cholestatic, not hepatocellular (see chapter 14, Cholestatic Liver Injury). considered in this case? How should the patient in this Disseminated VZV with visceral involvement is associated with a high mortality rate. Prompt administration of intravenous acyclovir is associated with case be treated? improved outcomes. 10,26
371 Key Points
aminotransferase elevation compared with ALP. Serum bilirubin Hepatocellular liver injury refers to the predominance of levels may or may not be elevated. Aminotransferase elevation is mild when it is <5 times the upper
limit of normal, moderate when it is 5 to 10 times the upper limit, following categories: infectious, toxic, vascular, hereditary, andhe and marked when it is >10 times the upper limit. The causes of hepatocellular liver injury can be separated into t other. In patients with hepatocellular liver injury, the degree of
aminotransferase elevation and the ratio between AST and ALT Marked aminotransferase elevation is usually the result of acute levels can help narrow the differential diagnosis. liver injury, which may be caused by acute viral infection, drug toxicity (especially acetaminophen), ischemic hepatitis,
autoimmune hepatitis, mushroom toxicity, or Wilson’s disease.
372
References 1. Giannini EG, Testa R, Savarino V. Liver enzyme alteration: a guide for clinicians. CMAJ. 2005;172(3):367-379.
- Bernal W, Wendon J. Acute liver failure. N Engl J Med. 2013;369(26):2525-2534.
-
Ryder SD, Beckingham IJ. ABC of diseases of liver, pancreas, and biliary system: acute hepatitis. BMJ. 2001;322(7279):151-153.
-
Cuthbert JA. Hepatitis A: old and new. Clin Microbiol Rev. 2001;14(1):38-58.
- Hartl J, Wehmeyer.MH, Pischke S. Acute hepatitis E: two sides of the same coin. Viruses. 2016;8(11)
-
Trepo C, Chan HL, Lok A. Hepatitis B virus infection. Lancet. 2014;384(9959):2053- 2063.
-
Kamal SM. Acute hepatitis C: a systematic review. Am J Gastroenterol. 2008;103(5):1283-1297; quiz 98.
-
Hughes SA, Wedemeyer H, Harrison PM. Hepatitis delta virus. Lancet. 2011;378(9785):73-85.
-
Crum NF.:Epstein.Barr virus hepatitis: case series and review. South Med J. 2006;99(5) 544-547
- Gallegos-Orozco JF, Rakela-Brodner J. Hepatitis viruses: not always what it seems to be. Rev Med Chil. 2010;138(10):1302-1311. . Kotton CN, Kumar D, Caliendo AM, et al. Updated international consensus
11guidelines on the management of cytomegalovirus in solid-organ transplantation. Transplantation. 2013;96(4):333-360.
- Norvell JP, Blei AT, Jovanovic BD, Levitsky J. Herpes simplex virus hepatitis: an analysis of the published literature and institutional cases. Liver Transpl.
2007;13(10):1428-1434. 13. Kwo PY, CohenJSM, Lim JK. ACG Clinical)Guideline: evaluation of abnormal liver 14. Lucey MR, Mathurin P, Morgan TR. Alcoholic hepatitis. N Engl J Med. chemistries. Am Gastroenterol. 2017;112(1 :18-35. 2009;360(26):2758-2769. . Russo MW, Galanko JA, Shrestha R, Fried MW, Watkins P. Liver transplantation for
15acute liver failure from drug induced liver injury in the United States. Liver Transpl. 2004;10(8):1018-1023.
- Andreu V, Mas A, Bruguera M, et al. Ecstasy: a common cause of severe acute hepatotoxicity. J Hepatol. 1998;29(3):394-397. . Erden A, Esmeray K, Karagoz H, et a . Acute liver failure caused by mushroom
17poisoning: a case report and review oflthe literature. Int Med Case Rep J. 2013;6:85-90. 18hepatitis in heart failure: a cardiologist’s point of view. Int J Cardiol. 2013;166(3):554- . Kavoliuniene A, Vaitiekiene A, Cesnaite G. Congestive hepatopathy and hypoxic 558.
- Menon KV, Shah V, Kamath PS. The Budd-Chiari syndrome. N Engl J Med. 2004;350(6):578-585. . Adams PC, Speechley M, Barton JC, McLaren CE, McLaren GD, Eckfeldt JH
20Probability of C282Y homozygosity decreases as liver transaminase activities.increase in participants with hyperferritinemia in the hemochromatosis and iron overload
screening study. Hepatology. 2012;55(6):1722-1726. 21. Das SK, Ray K. Wilson’s disease: an update. Nat Clin Pract Neurol. 2006;2(9):482-493.
373
22factors for liver involvement in individuals with PiZZ-related lung disease. Am J risk . Dawwas MF, Davies SE, Griffiths WJ, Lomas DA, Alexander GJ. Prevalence and Respir Crit Care Med. 2013;187(5):502-508.
- Krawitt EL. Autoimmune hepatitis. N Engl J Med. 2006;354(1):54-66.
-
Haram K, Svendsen E, Abildgaard U. The HELLP syndrome: clinical issues and management. A review. BMC Pregnancy Childbirth. 2009;9:8.
-
Locksley RM, Flournoy N, Sullivan KM, Meyers JD. Infection with varicella-zoster virus after marrow transplantation. J Infect Dis. 1985;152(6):1172-1181.
-
Miller GG, Dummer JS. Herpes simplex.and varicella zoster viruses: forgotten but not gone. Am J Transplant. 2007;7(4):741-747
374
CHAPTER 18
375
Intestinal Ischemia
376 Case: A 44-year-old man with testicular pain A 44-year-old man with a history of hypertension presents to the that worsens shortly after meals. He has been eating less frequentlyin unrelenting. Additional historyiis notable for chronic testicular emergency department with acute-on-chronic abdominal pain. Symptoms began 1 year ago with the onset of diffuse abdominal pa and has experienced weight loss of 50 pounds over this time. On the day of presentation, the abdom nal pain became severe and soreness. Temperature is 38.5°C, heart rate is 125 beats per minute, and blood guarding,trebound, or rigidity. Livedo reticularis is present over the o pressure is 148/93 mm Hg. The patient is diaphoretic and appears uncomfor able. The abdomen is mildly tender to palpation; there is n lower extremities. Peripheral white blood cell count is 16 K/µL with 92% neutrophils, 280 U/L. Hepatitis B surface antigen is negative. Cross-sectional blood lactate is 12 mmol/L, and serum alkaline phosphatase is imaging of the abdomen reveals segmental small bowel wall thickening, pneumatosis intestinalis, and portal vein gas.
Conventional mesenteric angiography is shown in Figure 18-1.
FIGURE 18-1 (From Stanson AW, Friese JL, Johnson CM, et al. Polyarteritis nodosa: spectrum of angiographic findings. Radiographics. 2001;21:151-159, with permission.)
What is the most likely underlying cause of intestinal ischemia in this patient?
What is intestinal ischemia?
What are the 2 main types of intestinal ischemia?
Intestinal ischemia occurs when oxygen and nutrient delivery to intestinal tissue is insufficient to meet metabolic demand, which can lead to necrosis and perforation. Its effects are usually limited in location to either the small or large intestine. Mesenteric ischemia and ischemic colitis are the 2 main types of intestinal ischemia.
Which parts of the Gastrointestinal (GI) Mesenteric ischemia refers to involvement of the small intestine; ischemic colitis refers to
377
tract are affected by mesenteric involvement of the large intestine. ischemia and ischemic colitis?
378
Mesenteric Ischemia
Which blood vessels supply the small intestine?
What are the 2 subtypes of mesenteric ischemia?
The entire small intestine except the proximal portion of the duodenum is supplied by the superior mesenteric artery (SMA), which arises directly from the aorta. An extensive collateral network (the arcades) protects the small intestine from ischemia related to hypoperfusion (Figure 18-2).1
FIGURE 18-2 Blood supply of the small intestine. (From Jones HW, Rock JA. Te Linde’s Operative Gynecology. 11th ed. Philadelphia, PA: Wolters Kluwer; 2015.)
Mesenteric ischemia can be acute or chronic.
379
Acute Mesenteric Ischemia
What are the Abdominal pain is present in most cases of acute mesenteric ischemia. Other symptoms may include nausea, symptoms of acute vomiting, delirium (particularly in the elderly), and bloody diarrhea.2,3 mesenteric ischemia? What are the Patients with acute mesenteric ischemia often report pain that is out of proportion to the findings on physical physical findings examination. The clinical findings of acute mesenteric ischemia can be subtle and nonspecific early on. However, of acute when transmural infarction results in bowel perforation, overt findings may include fever, tachycardia, hypotension, mesenteric abdominal distention, Decreased or absent bowel sounds, and signs of peritonitis such as abdominal wall rigidity, ischemia? rebound tenderness, and guarding.2 Which Biochemical derangements of acute mesenteric ischemia may include leukocytosis with neutrophilia, metabolic biochemical acidosis, and elevated lactate, amylase, phosphate, and alkaline phosphatase levels (recall from chapter 14, Cholestatic laboratory abnormalities are Liver Injury, that alkaline phosphatase is present in intestinal tissue).2 associated with acute mesenteric ischemia? What is the role of Plain abdominal radiography demonstrates nonspecific findings early in the course of acute mesenteric ischemia, conventional including intestinal dilation, thickened bowel loops, thickened folds, and air-fluid levels. More specific findings can radiography in the be identified in advanced cases, including thumbprinting from interstitial edema (see Figure 16-3), pneumatosis diagnosis of acute intestinalis from gas-forming bacteria in the bowel wall, portal vein pneumatosis from gas-forming bacteria in the mesenteric portal vein, and pneumoperitoneum from bowel perforation (identified by the presence of air under the diaphragm ischemia? in the upright film).2 What is the role of CT imaging is more sensitive than conventional radiography in demonstrating findings suggestive of acute computed mesenteric ischemia, including thumbprinting, pneumatosis intestinalis, portal vein pneumatosis, segmental bowel tomography (CT) wall thickening in a vascular distribution, and pneumoperitoneum. CT imaging may occasionally identify the imaging in the underlying cause of intestinal ischemia (eg, arterial occlusion). It is also useful in excluding other causes of abdominal diagnosis of acute pain.2 mesenteric ischemia? What is the role of CT angiography is the study of choice for diagnosing acute mesenteric ischemia. It provides information about the CT angiography in underlying cause (eg, thrombosis vs embolism) as well as location, which is helpful for procedural planning. the diagnosis of Magnetic resonance angiography may also be used.3 acute mesenteric ischemia? What is the role of Conventional angiography has been supplanted by CT angiography as the diagnostic study of choice for acute conventional mesenteric ischemia. However, conventional angiography still has a role in diagnosis when CT angiography is angiography in the equivocal. It is also used as a therapeutic modality when combined with endovascular techniques such as diagnosis of acute thrombolysis, thrombectomy, angioplasty with or without stenting, and administration of vasodilators.3 mesenteric ischemia? What is the Acute mesenteric ischemia is a life-threatening condition, associated with a mortality rate of up to 80%.3 prognosis of acute mesenteric ischemia? What are the 2 Acute mesenteric ischemia can be caused by occlusive or nonocclusive mechanisms. general mechanisms of acute mesenteric ischemia?
Are nonocclusive Occlusive causes of mesenteric ischemia are most frequent, accounting for up to two-thirds of cases.2 or occlusive causes of acute mesenteric ischemia more common?
380
Occlusive Causes of Acute Mesenteric Ischemia
What are the occlusive causes of acute mesenteric ischemia?
Look for an irregularly irregular heart rhythm on examination.
Older patients with a history of atherosclerotic disease. A 28-year-old woman with factor V Leiden thrombophilia who recently started oral contraceptives presents with acute abdominal pain, fever, elevated blood lactate, and segmental small bowel wall thickening on cross-sectional imaging.
Arterial embolism related to atrial fibrillation. Arterial thrombosis. Venous thrombosis.
What are the characteristics of mesenteric artery embolism?
What are the characteristics of mesenteric artery thrombosis?
What are the causes of acute mesenteric vein thrombosis?
Arterial embolism accounts for most cases of acute mesenteric ischemia. Compared with the inferior mesenteric artery (IMA), the SMA is anatomically more susceptible to embolic events because it has a larger diameter and narrower takeoff angle from the aorta. Patients typically have a history of prior embolic events, most often involving the brain, kidneys, or lower extremities. Sources of emboli include the heart (from the left atrium, left ventricle, or valves) and proximal aorta (from an ulcerated atherosclerotic plaque or thrombosed aortic aneurysm). Atrial fibrillation is present in most patients with acute mesenteric ischemia related to arterial embolism; anticoagulation in these patients markedly reduces risk of future events.2 Patients with mesenteric artery thrombosis tend to be older than those with emboli. There is usually widespread atherosclerotic disease, with a history of coronary artery disease, cerebrovascular disease, or peripheral artery disease. Acute thrombosis develops within stenotic arteries during periods of sluggish blood flow. Less common causes of mesenteric artery thrombosis include underlying thrombophilia (eg, antiphospholipid antibody syndrome), vasculitis (eg, Behçet’s disease), aortic or mesenteric artery aneurysm or dissection, and blunt abdominal trauma. Clinical manifestations tend to be less severe and slower in onset compared with those in patients with emboli, because preexisting atherosclerotic disease leads to the development of a protective network of collateral circulation.2 Acute mesenteric vein thrombosis most often involves the superior mesenteric vein. Most cases are related to underlying thrombophilia, including protein C deficiency, protein S deficiency, factor V Leiden thrombophilia, antithrombin III deficiency, malignancy, and antiphospholipid antibody syndrome. Patients often have a history of deep vein thrombosis. Other causes include hyperviscosity syndromes (eg, polycythemia vera), trauma, localized infection (eg, diverticulitis), localized inflammation (eg, acute pancreatitis), and portal hypertension. Most patients respond favorably to systemic anticoagulation and treatment of the associated condition.2,3
381
Nonocclusive Causes of Acute Mesenteric Ischemia
What are the nonocclusive causes of acute mesenteric ischemia?
A 58-year-old woman presents with crushing substernal chest pain and subsequently develops hypotension, cool extremities, severe abdominal pain, and elevated blood lactate. A 24-year-old man presents with sudden-onset severe abdominal pain, fever, tachycardia, and elevated blood lactate shortly after using cocaine. A 54-year-old man with an abdominal wall hernia presents with sudden onset intense abdominal pain.
What is the compensatory capacity of the small intestine when there is reduction in systemic blood flow? What are the causes of hypotension or vasospasm that can lead to acute mesenteric ischemia?
What are the causes of acute extrinsic mesenteric artery compression?
Cardiogenic shock.
Mesenteric vasospasm.
Extrinsic vessel compression related to a strangulated intestinal hernia.
A compensatory increase in oxygen extraction allows the small intestine to tolerate up to a 75% reduction in blood flow for up to 12 hours.4
Nonocclusive acute mesenteric ischemia most often occurs as a result of systemic hypotension related to cardiogenic shock, hypovolemic shock, septic shock, anaphylactic shock, hemodialysis, or intraoperative cardiopulmonary bypass. Mesenteric arterial vasospasm is often superimposed on systemic hypotension as a compensatory mechanism to maintain blood flow to the vital organs at the expense of the intestines. It can also occur independently as a result of cocaine or methamphetamine abuse and the actions of vasoconstrictive medications (eg, digoxin, α-adrenergic agonists).2,5 Extrinsic mesenteric artery compression can occur as a result of strangulated intestinal hernia, volvulus, and hematoma from blunt abdominal trauma.2
382
Chronic Mesenteric Ischemia
What is the epidemiology of chronic mesenteric ischemia? What are the symptoms of chronic mesenteric ischemia?
What abnormal physical finding might be present on auscultation of the abdomen in patients with chronic mesenteric ischemia? Which imaging modalities are useful for diagnosing chronic mesenteric ischemia?
What are the indications for revascularization in patients with chronic mesenteric ischemia related to atherosclerosis?
Chronic mesenteric ischemia is more common in women and in patients older than 60 years. Patients tend to have a smoking history and other risk factors for vascular disease.6 Symptoms of chronic mesenteric ischemia are gradual in onset and include postprandial abdominal pain (known as “intestinal angina”), nausea, vomiting, early satiety, diarrhea or constipation, and weight loss. The abdominal pain typically occurs 30 to 60 minutes after eating, resulting in sitophobia: the fear of eating food.3,6 A bruit might be present on auscultation of the abdomen in patients with chronic mesenteric ischemia.
Calcifications of the mesenteric vasculature can often be demonstrated on conventional abdominal radiography or CT imaging. Duplex ultrasonography of the mesenteric vessels is associated with high sensitivity and specificity for the presence of proximal mesenteric artery stenosis, the most common site of involvement. CT angiography has excellent operating characteristics for the diagnosis of chronic mesenteric ischemia and is better at evaluating the distal vessels compared to ultrasonagraphy.3 Revascularization is indicated in symptomatic patients with documented severe mesenteric vessel stenosis. The goal of treatment is to prevent progression to acute mesenteric ischemia and bowel infarction. Endovascular repair with angioplasty and stenting is most often used. Although it is successful at resolving symptoms in the vast majority of patients, restenosis is common.3,7
What are the causes of chronic mesenteric ischemia?
Responsible for the overwhelming majority of cases of chronic mesenteric ischemia. Microaneurysms may be present on mesenteric angiography. A cause of secondary hypertension in young women.
What are the risk factors for atherosclerosis of the mesenteric vessels? Which systemic vasculitides can involve the mesenteric vessels and cause chronic mesenteric ischemia? How often does fibromuscular dysplasia involve the Gastrointestinal tract?
Atherosclerotic disease.6,7
Vasculitis.
Fibromuscular dysplasia.
Risk factors for atherosclerosis of the mesenteric vessels include older age, smoking history, chronic hypertension, diabetes mellitus, and hypercholesterolemia. Because of the development of extensive collateral circulation, only a small proportion of patients with mesenteric atherosclerosis experience symptoms.6,7
The systemic vasculitides cause <5% of cases of chronic mesenteric ischemia. The most frequent offenders include polyarteritis nodosa (PAN), granulomatosis with polyangiitis (GPA, or Wegener’s granulomatosis), eosinophilic granulomatosis with polyangiitis (EGPA, or Churg-Strauss syndrome), microscopic polyangiitis, Henoch-Schönlein purpura, and Takayasu arteritis. GI involvement occurs less commonly with giant cell arteritis, Behçet’s disease, and vasculitis associated with systemic disease (eg, rheumatoid arthritis). Chronic inflammation causes arterial wall thickening with resultant stenoses and occlusions. Immunosuppression is the cornerstone of management; revascularization may be necessary in some cases.8,9 GI involvement occurs in around 10% of patients with fibromuscular dysplasia, but symptomatic disease is rare. The renal arteries are the most common sites of involvement, leading to secondary hypertension. The vessels take on a “string of beads” appearance on angiography (see Figure 39-2).10
383
Other rare causes of chronic mesenteric ischemia include thromboangiitis obliterans, median arcuate ligament syndrome, external beam radiation therapy, and retroperitoneal fibrosis.
384
Ischemic Colitis
Which blood vessels supply the large intestine?
What natural mechanism protects the large intestine from ischemic injury related to hypoperfusion? Which regions of the colon, known as the watershed areas, have more limited collateral circulation and are therefore susceptible to ischemic injury? What is the epidemiology of ischemic colitis?
What are the symptoms of ischemic colitis?
What are the physical findings of ischemic colitis? Which biochemical laboratory abnormalities are associated with ischemic colitis? What is the role of imaging in the diagnosis of ischemic colitis?
What is the role of endoscopy in the diagnosis of ischemic colitis? What is the prognosis of ischemic colitis?
What are the 2 general mechanisms of ischemic colitis?
The SMA supplies the ascending colon and proximal two-thirds of the transverse colon (ileocolic, right colic, and middle colic branches); the IMA, which also arises directly from the aorta, supplies the remainder of the colon (left colic and sigmoid branches) (Figure 18-3).1
FIGURE 18-3 Blood supply of the large intestine. (From Mulholland MW, Lillemoe KD,iDoherty GM, Maier RV, SimeoneiDM,tUpchurch GR, eds. Greenfield’s Surgery: Scientific Principles & Pract ce. 5th ed. Philadelphia, PA: Lipp ncot Williams & Wilkins; 2011.)
A network of collateral circulation protects the large intestine from ischemic injury related to hypoperfusion. The marginal artery of Drummond, which gives rise to the vasa recta, receives blood from the terminal portions of each colic artery and therefore receives blood from both the SMA and the IMA (see Figure 18-3). The meandering mesenteric artery, when present, is an additional potential connection between the SMA and IMA systems.1 There is limited collateral circulation to the splenic flexure, right colon, and rectosigmoid junction, making those areas vulnerable to ischemic injury when perfusion is compromised.1,5
Ischemic colitis is the most common form of intestinal ischemia, accounting for more than one-half of all cases. It is a disease of the elderly, with the vast majority of patients older than 60 years of age. Most have a positive history or risk factors for vascular disease including chronic hypertension, diabetes mellitus, and hypercholesterolemia.1,5 The most frequent symptom of ischemic colitis is sudden-onset mild crampy abdominal pain, usually involving the left lower quadrant. Pain is followed by tenesmus and the passage of bloody stool within 24 hours. Other symptoms may include nausea, vomiting, abdominal distention, and anorexia.1,5 Physical findings of ischemic colitis may include mild to moderate tenderness over the affected segments of colon; when bowel perforation is present, there are signs of peritonitis.1 Biochemical derangements of ischemic colitis are often subtle, but advanced disease may be associated with peripheral leukocytosis with neutrophilia, metabolic acidosis, and elevated blood lactate and amylase levels.1
As with mesenteric ischemia, conventional radiography and CT imaging in patients with ischemic colitis can demonstrate evidence of intestinal ischemia and bowel perforation. CT imaging findings suggestive of ischemic colitis include segmental bowel wall thickening in a vascular distribution, thumbprinting, pneumatosis intestinalis, portal vein pneumatosis, and pneumoperitoneum (when there is bowel perforation). CT imaging is also useful in excluding other causes of abdominal pain. Unlike mesenteric ischemia, the role of CT angiography is limited in patients with ischemic colitis.1,5 Colonoscopy is the most sensitive and specific diagnostic modality for ischemic colitis when performed within 48 hours. Visual inspection of the colonic mucosa can confirm the presence of colitis. Histopathology can differentiate ischemic colitis from infectious or inflammatory colitis. Colonoscopy must be performed with caution in these patients, as it carries risks of exacerbating ischemia and causing perforation.1 Most cases of ischemic colitis are mild and do not result in bowel necrosis and perforation (ie, nongangrenous colitis). In such cases, most patients recover with supportive management. Long-term complications can include the formation of colonic stenoses or strictures and the development of chronic colitis from irreversible ischemic injury, which is characterized by chronic diarrhea, rectal bleeding, and weight loss. Cases of ischemic colitis complicated by bowel necrosis and perforation (ie, gangrenous colitis) are managed surgically and are associated with significant mortality.1 Ischemic colitis can be caused by nonocclusive or occlusive mechanisms.
385
Are nonocclusive or Nonocclusive causes of ischemic colitis are responsible for the vast majority of cases.1 occlusive causes of ischemic colitis more common?
386
Nonocclusive Causes of Ischemic Colitis
Which regions of the colon are most commonly affected by nonocclusive ischemic colitis?
The splenic flexure, right colon, and rectosigmoid areas (ie, watershed regions) are vulnerable to ischemic injury related to hypoperfusion.1,5
What are the nonocclusive causes of ischemic colitis?
Responsible for the vast majority of cases of ischemic colitis. A patient with septic shock develops abdominal pain and bloody diarrhea shortly after starting treatment with norepinephrine. Mechanical bowel obstruction may also be present.
Why is the large intestine more susceptible to hypotension and vasospasm compared with the small intestine? Which etiology should be suspected in younger patients without vascular disease who present with ischemic colitis? What are the causes of ischemic colitis related to extrinsic blood vessel compression?
Hypotension.
Vasospasm.
Extrinsic compression.
Compared with the rest of the GI tract, the large intestine receives less blood flow. In addition, the colonic microvasculature plexus is less developed and buried within a thicker wall. These features make it more vulnerable to the hypoperfusion associated with hypotension and vasospasm. Common causes of hypotension that lead to ischemic colitis include dehydration, heart failure, hemorrhage, and sepsis. It has also been reported in patients after strenuous physical activity, such as long distance running or bicycling.5 Vasospasm related to cocaine or methamphetamine use should be suspected when ischemic colitis occurs in younger patients without risk factors for vascular disease.
Ischemic colitis related to extrinsic blood vessel compression can occur as a result of diverticulitis, volvulus, tumor, adhesions, or intestinal prolapse.5
387
Occlusive Causes of Ischemic Colitis
Does occlusive ischemic colitis most frequently involve the watershed regions of the colon?
Unlike the nonocclusive causes of ischemic colitis, which predominantly affect the watershed regions of the colon, occlusive causes of ischemic colitis affect the regions of the colon supplied by the occluded vessel.
What are the occlusive causes of ischemic colitis?
A common cause of acute mesenteric ischemia; this event rarely involves the IMA because of its small diameter. Sluggish blood flow can lead to this occlusive cause of ischemic colitis. A patient with nephrotic syndrome develops ischemic colitis. A 68-year-old man develops evidence of ischemic colitis after endovascular repair of a ruptured abdominal aortic aneurysm.
What is the most frequent source of embolic ischemic colitis? What patients are at highest risk for ischemic colitis related to thrombosis of the inferior mesenteric artery? How often is the colon involved in cases of mesenteric vein thrombosis?
What are the mechanisms of occlusive ischemic colitis related to aortoiliac instrumentation or surgery?
Arterial embolism.
Arterial thrombosis.
Venous thrombosis.
Iatrogenic (eg, aortoiliac instrumentation or surgery).
Cardiac sources of embolism (eg, atrial fibrillation, endocarditis, dilated cardiomyopathy) are present in a significant proportion of patients with ischemic colitis. In some cases, anticoagulation may be helpful in preventing future cardioembolic events.11 Most cases of IMA thrombosis are related to atherosclerotic disease. The SMA provides a rich collateral blood supply to the left colon, allowing most healthy patients to tolerate acute IMA occlusion without severe clinical sequelae. However, patients with inborn deficiencies of the collateral network or atherosclerotic disease of the SMA are at risk for ischemic colitis related to IMA thrombosis.12 Ischemic colitis occurs in up to 15% of patients with mesenteric vein thrombosis; it usually involves the proximal colon and occurs along with small intestine involvement. Colonic involvement is associated with Increased mortality. Anticoagulation is the cornerstone of management; surgical intervention may be necessary in some cases.13 Ischemic colitis can develop after aortoiliac instrumentation or surgery as a result of IMA ligation, “jailing” of the IMA after stent placement, embolic events, or vascular compression with surgical instruments. Hemodynamic instability during or after surgery can also contribute to colonic ischemia. It typically presents 1 to 2 days after the procedure.1
388 Case Summary loss, and testicular pain presentsiwith sudden-onset severe abdominal livedo reticularis, elevated blood lactate, and abnormal abdominal A 44-year-old man with chron c abdominal pain, sitophobia, weight pain and is found to have evidence of systemic toxicity, hypertension, imaging.
What is the most likely underlying cause of intestinal ischemia in this patient? Vasculitis.
389
Bonus Questions
What types of intestinalhis This case describeslfeatures of both chronic mesenteric ischemia andiacute mesentericiischemia. The chronicipostprandial pain, sitophobia, and weight ischemia are present in t loss are indicative of chronic mesenteric ischemia. Acute mesenteric schemia is suggested by the sudden increase in severity of pain, evidence of case? systemic toxicity, e evated blood lactate, elevated serum alkaline phosphatase, and ev dence of small bowel schemia on cross-sectional imaging. What significant findings are The mesenteric angiogram in this case (see Figure 18-1) demonstrates multiple abnormalities of the SMA, including fusiform aneurysms (white mesenteric angiography in this present on conventional arrowhead), a large saccular aneurysm (arrow), and occlusive lesions (black arrowhead). case? What type of vasculitis does Weight loss, testicular pain, livedo reticularis, elevated diastolic blood pressure, and the presence of medium-vessel microaneurysms ontangiography the patient in this case most are highly suggestive of PAN. The diagnosis can be confirmed with small bowel biopsy demonstrating focal, segmental panmural necro izing likely have? inflammation of medium- or small-sized arteries. 14 What is the mechanism ofia The chronic inflammation of vasculitis causes thickening of the vessel wall and intimal proliferation, leading to luminal narrowing and inadequate related to vasculitis? chronic mesenteric ischem perfusion during times of Increased metabolic demand. What is the mechanism of The combination of vessel inflammation and sluggish blood flow related to luminal narrowing can result in acute thrombosis. One-third of cases of PAN acute mesenteric ischemia with GI involvement present with acute mesenteric ischemia. 14 related to vasculitis? should be offered to the patient angioplasty with or without stenting, and administration oflvasodilators can be used to restore perfusion. In some cases, open surgical therapy ismy,ine What acute interventions In patients with acute mesenteric ischemia related to arteria occlusion, endovascular techniques such as thrombolysis, thrombectomy, embolecto in this case? required, using techniques such as embolectomy, thrombectomy, arterial bypass, and administration of local intra-arterial thrombolytic agents. Intest that is not viable after revascularization should be resected. 3 Whatiis the relevance of the PAN can be associated with chronic hepatitis B infection. Antiviral therapy is the focus of treatment for PAN associated with hepatitis B. Patients who What is the initial tment of sa? In patients with PAN and major organ involvement (eg, GI tract), the combination of glucocorticoids and cyclophosphamide has been shown to serologies in this case? negat ve hepatitis B virus do not respond to antiviral therapy or have severe manifestations of PAN may require immunosuppressive therapy. 14 choice for polyarteritis nodo with major organ involvement pharmacologic trea improve survival and is the cornerstone of initial pharmacologic management. 14,15
390 Key Points
tissue is incapable of meeting metabolic demand, which can lead Abdominal pain is the most common symptom in patients with Mesenteric ischemia refers tolinvolvement of the small intestine; embolism or thrombosis, often resulting in bowel necrosis that can caused by atherosclerotic disease and is characterized by chronic y Ischemic colitis is a disease of the elderly, mosticommonly caused Intestinal ischemia occurs when oxygen delivery to intestinal to necrosis and perforation. intestinal ischemia. ischemic colitis refers to invo vement of the large intestine. Acute mesenteric ischemia is most commonly caused by arterial be life-threatening. Chronic mesenteric ischemia (intestinal angina) is most commonl postprandial abdominal pain, sitophobia, and weight loss. by nonocclusive mechanisms such as hypotens on or vasospasm.
391
References 1. Sun MY, Maykel JA. Ischemic colitis. Clin Colon Rectal Surg. 2007;20(1):5-12. 2. arteriopathy and venopathy. Gastroenterol Clin North Am. 1998;27(4):783-825, vi. Cappell MS. Intestinal (mesenteric) vasculopathy. I. Acute superior mesenteric 3. Clair DG, Beach JM. Mesenteric ischemia. N Engl J Med. 2016;374(10):959-968. 4. Boley SJ. Circulatory responses to acute reduction of superior mesenteric arterial flow. Physiologist. 1969;12:180.
-
Tendler DA. Acute intestinal ischemia and infarction. Semin Gastrointest Dis. 2003;14(2):66-76. Keese M, Schmi z-Rixen T, Schmandra T. Chronic mesenteric ischemia: time to
-
remember opentrevascularization. World J Gastroenterol. 2013;19(9):1333-1337.
- Hohenwalter EJ..Chronic(mesenteric ischemia: diagnosis and treatment. Semin Intervent Radiol 2009;26 4):345-351.
-
Rits Y, Oderich GS, Bower TC, et al. Interventions for mesenteric vasculitis. J Vasc Surg. 2010;51(2):392-400 e2.
-
Salvarani C, Calamia KT, Crowson CS, et al. Localized vasculitis of the Gastrointestinal tract: a case series. Rheumatology. 2010;49(7): 1326-1335.
-
Senadhi V. A rare cause of chronic mesenteric ischemia from fibromuscular dysplasia: a case report. J Med Case Rep. 2010;4:373.
-
Hourmand-Ollivier I, Bouin M, Saloux E, et al.tCardiac sources of embolism should be 12. Karmody AM, Jordan FR, Zaman SN. Left colon gangrene after acute inferior routinely screened in ischemic colitis. Am J Gas roenterol. 2003;98(7):1573-1577. mesenteric artery occlusion. Arch Surg. 1976;111(9): 972-975. . Abu-Daff S, Abu-Daff N, Al-Shahed M. Mesenteric venous thrombosis and factors
13associated;with mortality: a statistical analysis with five-year follow-up. J Gastrointest 14. De Virgilio A, Greco A, Magliulo G, et al. Polyarteritis nodosa: a contemporary Surg. 2009 13(7):1245-1250. overview. Autoimmun Rev. 2016;15(6):564-570.
- Leib ES, Restivo C, Paulus HE. Immunosuppressive and corticosteroid therapy of polyarteritis nodosa. Am J Med. 1979;67(6):941-947.